Site Loader

Содержание

Как доказать что векторы перпендикулярны

Ключевые слова:
перпендикулярность, прямая линия, параллельность, вектор, коллинеарные вектора, координаты

Условие перпендикулярности векторов

Условие коллинеарности векторов


См. также:
Углы на плоскости, Координаты вектора

Напомним определение перпендикулярных векторов на плоскости и в трехмерном пространстве.

Два ненулевых вектора называются перпендикулярными, если угол между ними равен девяноста градусам ( радиан).

Для перпендикулярности двух ненулевых векторов и необходимо и достаточно, чтобы их скалярное произведение равнялось нулю, то есть, чтобы выполнялось равенство .

Пусть векторы и перпендикулярны. Докажем выполнение равенства .

По определению скалярное произведение векторов равно произведению их длин на косинус угла между ними. Так как векторы и перпендикулярны, то угол между ними равен девяноста градусам, следовательно, , что и требовалось доказать.

Переходим ко второй части доказательства.

Теперь считаем, что . Докажем, что векторы и перпендикулярны.

Так как векторы и ненулевые, то из равенства следует, что . Таким образом, косинус угла между векторами и равен нулю, следовательно, угол равен , что указывает на перпендикулярность векторов и .

Итак, необходимое и достаточное условие перпендикулярности двух векторов полностью доказано.

Как же выглядит условие перпендикулярности двух векторов в координатной форме?

В разделе скалярное произведение в координатах мы показали, что для двух векторов с заданными координатами и на плоскости справедливо равенство , а для двух векторов и в пространстве . Таким образом,

необходимое и достаточное условие перпендикулярности двух векторов в координатах имеет вид на плоскости, а в трехмерном пространстве .

Рассмотрим применение полученных условий на практике, для этого разберем решение нескольких примеров.

Перпендикулярны ли векторы .

Вычислим их скалярное произведение по координатам . Следовательно, условие перпендикулярности двух векторов на плоскости выполнено, то есть, они перпендикулярны.

да, векторы перпендикулярны.

Перпендикулярны ли векторы и , где – координатные векторы прямоугольной системы координат в трехмерном пространстве.

Векторы и имеют соответственно координаты и (при необходимости смотрите статью координаты вектора в прямоугольной системе координат). Проверим выполнение необходимого и достаточного условия перпендикулярности двух векторов:

Так как , то векторы и не перпендикулярны.

нет, не перпендикулярны.

Найдите значение , при котором векторы и перпендикулярны.

Воспользуемся условием перпендикулярности двух векторов в пространстве в координатной форме

векторы перпендикулярны при .

В некоторых случаях возможно ответить на вопрос о перпендикулярности двух векторов без использования необходимого и достаточного условия перпендикулярности. Например, когда известны длины всех сторон треугольника, построенного на двух векторах, то можно найти угол между векторами и посмотреть, равен ли он девяноста градусам.

Стороны АВ, АС и ВС треугольника АВС равны соответственно 8, 6 и 10 см. Убедитесь, что векторы и перпендикулярны.

Если векторы и перпендикулярны, то треугольник АВС – прямоугольный и его гипотенузой является сторона ВС. Тогда по теореме Пифагора должно выполняться равенство . Проверим его справедливость: .

Следовательно, АВ и АС – катеты прямоугольного треугольника АВС, поэтому, векторы и перпендикулярны.

Не нашли то, что искали? Воспользуйтесь поиском:

Лучшие изречения:

Только сон приблежает студента к концу лекции. А чужой храп его отдаляет. 8828 – | 7538 – или читать все.

78.85.5.224 © studopedia.ru Не является автором материалов, которые размещены. Но предоставляет возможность бесплатного использования. Есть нарушение авторского права? Напишите нам | Обратная связь.

Отключите adBlock!
и обновите страницу (F5)

очень нужно

Данная статья раскрывает смысл перпендикулярности двух векторов на плоскости в трехмерном пространстве и нахождение координат вектора, перпендикулярному одному или целой паре векторов. Тема применима для задач, связанных с уравнениями прямых и плоскостей.

Мы рассмотрим необходимое и достаточное условие перпендикулярности двух векторов, решим по методу нахождения вектора, перпендикулярному заданному, затронем ситуации по отысканию вектора, который перпендикулярен двум векторам.

Необходимое и достаточное условие перпендикулярности двух векторов

Применим правило о перпендикулярных векторах на плоскости и в трехмерном пространстве.

При условии значения угла между двумя ненулевыми векторами равным 90 ° ( π 2 радиан) называют перпендикулярными.

Что это значит, и в каких ситуациях необходимо знать про их перпендикулярность?

Установление перпендикулярности возможно через чертеж. При отложении вектора на плоскости от заданных точек можно геометрически измерить угол между ними. Перпендикулярность векторов если и будет установлена, то не совсем точно. Чаще всего данные задачи не позволяют делать это при помощи транспортира, поэтому данный метод применим только в случае, когда ничего больше о векторах неизвестно.

Большинство случаев доказательства перпендикулярности двух ненулевых векторов на плоскости или в пространстве производится с помощью необходимого и достаточного условия перпендикулярности двух векторов.

Скалярное произведение двух ненулевых векторов a → и b → равном нулю для выполнения равенства a → , b → = 0 достаточно для их перпендикулярности.

Пусть заданные векторы a → и b → перпендикулярны, тогда выполним доказательство равенства a ⇀ , b → = 0 .

Из определения про скалярное произведение векторов мы знаем, что оно равняется произведению длин заданных векторов на косинус угла между ними. По условию a → и b → перпендикулярны, а, значит, исходя из определения, угол между ними 90 ° . векторов a → и b → равен 90 ° . По определению это и есть необходимое и достаточное свойство.

Условие перпендикулярности на координатной плоскости

Раздел скалярного произведения в координатах демонстрирует неравенство ( a → , b → ) = a x · b x + a y · b y , справедливое для векторов с координатами a → = ( a x , a y ) и b → = ( b x , b y ) , на плоскости и ( a → , b → ) = a x · b x + a y · b y для векторов a → = ( a x , a y , a z ) и b → = ( b x , b y , b z ) в пространстве. Необходимым и достаточным условием перпендикулярности двух векторов в координатной плоскости имеет вид a x · b x + a y · b y = 0 , для трехмерного пространства a x · b x + a y · b y + a z · b z = 0 .

Применим на практике и рассмотрим на примерах.

Проверить свойство перпендикулярности двух векторов a → = ( 2 , – 3 ) , b → = ( – 6 , – 4 ) .

Для решения данной задачи необходимо найти скалярное произведение. Если по условию оно будет равным нулю, значит, они перпендикулярны.

( a → , b → ) = a x · b x + a y · b y = 2 · ( – 6 ) + ( – 3 ) · ( – 4 ) = 0 . Условие выполнено, значит, заданные векторы перпендикулярны на плоскости.

Ответ: да, заданные векторы a → и b → перпендикулярны.

Даны координатные векторы i → , j → , k → . Проверить, могут ли векторы i → – j → и i → + 2 · j → + 2 · k → быть перпендикулярными.

Для того, чтобы вспомнить, как определяются координаты вектора, нужно прочитать статью про координаты вектора в прямоугольной системе координат. Таким образом получаем, что у заданных векторов i → – j → и i → + 2 · j → + 2 · k → имеются соответствующие координаты ( 1 , – 1 , 0 ) и ( 1 , 2 , 2 ) . Подставляем числовые значения и получаем: i → + 2 · j → + 2 · k → , i → – j → = 1 · 1 + ( – 1 ) · 2 + 0 · 2 = – 1 .

Выражение не равно нулю, ( i → + 2 · j → + 2 · k → , i → – j → ) ≠ 0 , а это означает, что векторы i → – j → и i → + 2 · j → + 2 · k → не перпендикулярны, так как условие не выполнилось.

Ответ: нет, векторы i → – j → и i → + 2 · j → + 2 · k → не перпендикулярны.

Даны векторы a → = ( 1 , 0 , – 2 ) и b → = ( λ , 5 , 1 ) . Найти значение λ , при котором данные векторы перпендикулярны.

Используем условие перпендикулярности двух векторов в пространстве в квадратной форме, тогда получим

a x · b x + a y · b y + a z · b z = 0 ⇔ 1 · λ + 0 · 5 + ( – 2 ) · 1 = 0 ⇔ λ = 2

Ответ: векторы перпендикулярны при значении λ = 2 .

Имеются случаи, когда вопрос о перпендикулярности невозможен даже при необходимом и достаточном условии. При известных данных о трех сторонах треугольника на двух векторах, возможно, найти угол между векторами и проверить его.

Дан треугольник А В С со сторонами А В = 8 , А С = 6 , В С = 10 см. проверить на перпендикулярность векторы A B → и A C → .

При перпендикулярности векторов A B → и A C → треугольник A B C считается прямоугольным. Тогда применим теорему Пифагора, где В С – гипотенуза треугольника. Равенство B C 2 = A B 2 + A C 2 должно выполниться. Отсюда следует, что 10 2 = 8 2 + 6 2 ⇔ 100 = 100 . Значит, А В и А С являются катетами треугольника А В С , следовательно, A B → и A C → перпендикулярны.

Нахождение вектора, перпендикулярного данному

Важно научиться находить координаты вектора, перпендикулярного заданному. Это возможно как на плоскости, так и в пространстве при условии перпендикулярности векторов.

Нахождение вектора, перпендикулярного данному в плоскости.

Ненулевой вектор a → может иметь бесконечное количество перпендикулярных векторов на плоскости. Изобразим это на координатной прямой.

Задан ненулевой вектор a → , лежащий на прямой а. Тогда заданный b → , расположенный на любой прямой, перпендикулярной прямой а, становится перпендикулярным и a → . Если вектору i → перпендикулярен вектор j → или любой из векторов λ · j → при λ равной любому действительному числу кроме нуля, то нахождение координат вектора b → , перпендикулярному a → = ( a x , a y ) , сводится к бесконечному множеству решений. Но необходимо найти координаты вектора, перпендикулярного a → = ( a x , a y ) . Для этого необходимо записать условие перпендикулярности векторов в такой форме a x · b x + a y · b y = 0 . Имеем b x и b y , являющиеся искомыми координатами перпендикулярного вектора. Когда a x ≠ 0 , значение b y является ненулевым, а b x вычислим из неравенства a x · b x + a y · b y = 0 ⇔ b x = – a y · b y a x . При a x = 0 и a y ≠ 0 присваиваем b x любое значение кроме нуля, а b y находим из выражения b y = – a x · b x a y .

Дан вектор с координатами a → = ( – 2 , 2 ) . Найти перпендикулярный данному вектор.

Обозначим искомый вектор как b → ( b x , b y ) . Найти его координаты можно из условия перпендикулярности векторов a → и b → . Тогда получим: ( a → , b → ) = a x · b x + a y · b y = – 2 · b x + 2 · b y = 0 . Присвоим b y = 1 и подставим: – 2 · b x + 2 · b y = 0 ⇔ – 2 · b x + 2 = 0 . Отсюда из формулы получим b x = – 2 – 2 = 1 2 . Значит, вектор b → = ( 1 2 , 1 ) является вектором, перпендикулярным a → .

Если ставится вопрос о трехмерном пространстве, задача решается по такому же принципу. При заданном векторе a → = ( a x , a y , a z ) существует бесконечное множество перпендикулярных векторов. Зафиксирует это на координатной трехмерной плоскости. Дана a → , лежащая на прямой a . Перпендикулярную прямой a плоскость обозначаем α . В этом случае любой ненулевой вектор b → из плоскости α перпендикулярен a → .

Необходимо найти координаты b → , перпендикулярного ненулевому вектору a → = ( a x , a y , a z ) .

Пусть задан b → с координатами b x , b y и b z . Чтобы найти их, необходимо применить определение условия перпендикулярности двух векторов. Равенство a x · b x + a y · b y + a z · b z = 0 должно выполняться. Из условия a → – ненулевой, значит, одна из координат имеет значение не равное нулю. Предположим, что a x ≠ 0 , ( a y ≠ 0 или a z ≠ 0 ). Следовательно, имеем право разделить на эту координату все неравенство a x · b x + a y · b y + a z · b z = 0 , получим выражение b x + a y · b y + a z · b z a x = 0 ⇔ b x = – a y · b y + a z · b z a x . Присваиваем координатам b y и b x любое значение, вычисляем значение b x , исходя из формулы, b x = – a y · b y + a z · b z a x . Искомый перпендикулярный вектор будет иметь значение a → = ( a x , a y , a z ) .

Рассмотрим доказательство на примере.

Дан вектор с координатами a → = ( 1 , 2 , 3 ) . Найти вектор, перпендикулярный данному.

Обозначим искомый вектор за b → = ( b x , b y , b z ) . Исходя из условия о перпендикулярности векторов, скалярное произведение должно быть равным нулю.

a ⇀ , b ⇀ = 0 ⇔ a x · b x + a y · b y + a z · b z = 0 ⇔ 1 · b x + 2 · b y + 3 · b z = 0 ⇔ b x = – ( 2 · b y + 3 · b z )

Если значение b y = 1 , b z = 1 , тогда b x = – 2 · b y – 3 · b z = – ( 2 · 1 + 3 · 1 ) = – 5 . Отсюда следует, что координаты вектора b → ( – 5 , 1 , 1 ) . Вектор b → является одним из перпендикулярных векторов заданному.

Ответ: b → = ( – 5 , 1 , 1 ) .

Нахождение координат вектора, перпендикулярного двум заданным векторам

Нужно найти координаты вектора в трехмерном пространстве. Он перпендикулярен не коллинеаренным векторам a → ( a x , a y , a z ) и b → = ( b x , b y , b z ) . При условии коллинеарности векторов a → и b → в задаче достаточно будет найти вектор, перпендикулярный a → или b → .

При решении применяется понятие векторного произведения векторов.

Векторным произведением векторов a → и b → называют вектор, одновременно перпендикулярный и a → и b → . Для решения данной задачи применяется векторное произведение a → × b → . Для трехмерного пространства имеет вид a → × b → = a → j → k → a x a y a z b x b y b z

Разберем подробнее векторное произведение на примере задачи.

Заданы векторы b → = ( 0 , 2 , 3 ) и a → = ( 2 , 1 , 0 ) . Найти координаты любого перпендикулярного вектора данным одновременно.

Для решения необходимо найти векторное произведение векторов. (Необходимо обратиться к пункту вычисления определителя матрицы для нахождения вектора). Получим :

a → × b → = i → j → k → 2 1 0 0 2 3 = i → · 1 · 3 + j → · 0 · 0 + k → · 2 · 2 – k → · 1 · 0 – j → · 2 · 3 – i → · 0 · 2 = 3 · i → + ( – 6 ) · j → + 4 · k →

Ответ: ( 3 , – 6 , 4 ) координаты вектора, одновременно перпендикулярного заданным a → и b → .

Нормаль — Википедия с видео // WIKI 2

Пользуясь координатами какого-либо вектора на плоскости xOy, получим координаты двух равных по модулю векторов, перпендикулярных первоначальному вектору

Необходимое и достаточное условие перпендикулярности двух векторов.

Напомним определение перпендикулярных векторов на плоскости и в трехмерном пространстве.

Определение.

Два ненулевых вектора называются перпендикулярными, если угол между ними равен девяноста градусам ( радиан).

Какие же мысли навевает это определение, когда нужно узнать, перпендикулярны два изображенных вектора или нет?

Если от некоторой точки плоскости отложить векторы равные заданным векторам, то с помощью транспортира можно измерить угол между ними. Это позволит с некоторой степенью точности установить перпендикулярность векторов (если при измерении получили угол девяносто градусов). При этом конечно же следует учитывать точность построения и точность измерения. Такой метод для определения перпендикулярности двух векторов следует использовать только тогда, когда мы ничего не знаем об этих векторах, а имеем только их изображение на плоскости.

На практике часто приходится доказывать перпендикулярность двух ненулевых векторов, когда известны их координаты в прямоугольной системе координат на плоскости или в пространстве. В этом случае используется необходимое и достаточное условие перпендикулярности двух векторов. Сформулируем его в виде теоремы.

Теорема.

Для перпендикулярности двух ненулевых векторов и необходимо и достаточно, чтобы их скалярное произведение равнялось нулю, то есть, чтобы выполнялось равенство .

Доказательство.

Пусть векторы и перпендикулярны. Докажем выполнение равенства .

По определению скалярное произведение векторов равно произведению их длин на косинус угла между ними. Так как векторы и перпендикулярны, то угол между ними равен девяноста градусам, следовательно, , что и требовалось доказать.

Переходим ко второй части доказательства.

Теперь считаем, что . Докажем, что векторы и перпендикулярны.

Так как векторы и ненулевые, то из равенства следует, что . Таким образом, косинус угла между векторами и равен нулю, следовательно, угол равен , что указывает на перпендикулярность векторов и .

Итак, необходимое и достаточное условие перпендикулярности двух векторов полностью доказано.

Как же выглядит условие перпендикулярности двух векторов в координатной форме?

В разделе скалярное произведение в координатах мы показали, что для двух векторов с заданными координатами и на плоскости справедливо равенство , а для двух векторов и в пространстве . Таким образом, необходимое и достаточное условие перпендикулярности двух векторов в координатах имеет вид на плоскости, а в трехмерном пространстве .

Рассмотрим применение полученных условий на практике, для этого разберем решение нескольких примеров.

Пример.

Перпендикулярны ли векторы .

Решение.

Вычислим их скалярное произведение по координатам . Следовательно, условие перпендикулярности двух векторов на плоскости выполнено, то есть, они перпендикулярны.

Ответ:

да, векторы перпендикулярны.

Пример.

Перпендикулярны ли векторы и , где – координатные векторы прямоугольной системы координат в трехмерном пространстве.

Решение.

Ответ:

нет, не перпендикулярны.

Пример.

Найдите значение , при котором векторы и перпендикулярны.

Решение.

Воспользуемся условием перпендикулярности двух векторов в пространстве в координатной форме

Ответ:

векторы перпендикулярны при .

В некоторых случаях возможно ответить на вопрос о перпендикулярности двух векторов без использования необходимого и достаточного условия перпендикулярности. Например, когда известны длины всех сторон треугольника, построенного на двух векторах, то можно найти угол между векторами и посмотреть, равен ли он девяноста градусам.

Пример.

Стороны АВ, АС и ВС треугольника АВС равны соответственно 8, 6 и 10 см. Убедитесь, что векторы и перпендикулярны.

Решение.

Источник: http://cleverstudents.ru/vectors/condition_of_vectors_perpendicularity.html

Координаты вектора на плоскости, равного по модулю и перпендикулярного данному

Пусть на плоскости заданы координаты какого-либо вектора. Из этих координат получим координаты двух дополнительных векторов, перпендикулярных первоначальному вектору. Все три вектора будут иметь равные длины и располагаться в плоскости xOy.

Алгоритм получения координат перпендикулярных векторов

Вектор на плоскости xOy, перпендикулярный данному вектору получают так:

  1. Поменять местами координатные числа «x» и «y».
  2. Заменить знак у одной из координат на противоположный.

Графический пример

Рассмотрим небольшой графический пример (рис. 1).

Рис. 1. На рисунке векторы, обозначенные черным цветом, перпендикулярны вектору, обозначенному красным цветом

На плоскости проведены три вектора: один красный и два черных и, отмечены их координаты. Рассмотрим подробнее координаты двух векторов: (vec{a}) и (vec{b}).

[ vec{a} = left{ 4 ; 3 right} ]

[ vec{b} = left{ -3 ; 4 right} ]

Из рисунка видно, что векторы (vec{a}) и (vec{b}) перпендикулярны: ( vec{a} perp vec{b} ).

Вектор ( -vec{b} = left{ 3 ; -4 right} ), также будет перпендикулярным вектору ( vec{a} ): ( vec{a} perp vec{(-b)} )

Векторы, изображенные черным цветом, перпендикулярны красному вектору.

Длины векторов ( vec{a} ), ( vec{b} ) и ( vec{(-b)} ) равны.

Источник: http://formulki.ru/vektory/perpendikulyarnost-vektorov

1 Введение

Векторына плоскости и в пространстве, векторные поля используются не тольков математике, но и в физике, например, в электродинамике. Векторыпозволяют эффектно решать геометрические задачи второйчасти ЕГЭ и аналитической геометрии в пространстве.

Векторыв пространстве включают геометрия 10, 11 класса, аналитическаягеометрия. Векторы в пространстве даются аналогично векторам наплоскости, но учитывается третья координата z.Исключение из векторов в пространстве третьего измерения дает векторына плоскости, которые изучаются в геометрии 8, 9 класса.

Источник: http://super-code.ru/vektor-1/vektor-1.html

Функции как векторы

Большая часть этой статьи будет рассматривать функции, которые принимают и возвращают векторы различных типов. Чтобы её понять, нужно совершить некоторый умственный кувырок, который может показаться контринтуитивным, если вы не встречали его ранее.

Вот он: функции, которые возвращают векторы, сами являются векторами

С первого взгляда это утверждение может показаться бессмысленным. Векторы и функции — это совершенно разные вещи, как например яблоки и… стулья, не так ли? Как функция буквально может быть вектором?

Посмотрев на формальное определение векторного пространства, вы не найдёте в нем ничего конкретного о структуре векторов. Мы часто представляем их себе как стрелки с длиной и направлением, или как упорядоченные наборы чисел (координат). Но всё же, в сухом остатке для создания векторного пространства нужен набор сущностей, которые поддерживают две базовые операции: сложение между собой и умножение на скаляры (здесь это действительные числа). Эти операции должны подчиняться нескольким разумным аксиомам.

Функции тоже можно складывать между собой! Две функции и можно сложить поточечно и получить новую функцию для каждой точки в области определения. Аналогично, функция может быть поточечно умножена на скаляр: . Эти операции удовлетворяют аксиомам векторного пространства, потому любое множество подходящих функций формирует полноправное векторное пространство, называемое функциональным пространством.

Запишем формальнее: пусть есть область определения (любое множество, не обязательно векторное пространство) и область значений — векторное пространство. Тогда множество функций образуют векторное пространство относительно поточечного сложения и умножения на скаляр. Область значений должна быть векторным пространством чтобы в нём работало сложение и перемножение значений функции, но области определения не обязательно быть векторным пространством, и вообще “пространством”. Она может быть хоть дискретным множеством.

Понимание трактовки функций как векторов даёт возможность применять методы линейной алгебры при работе в функциями. Получается большая ветвь математики, называемая функциональным анализом.

Источник: http://habr.com/ru/post/529978/

2 Вектор на плоскости и в пространстве

Векторомназывается направленный отрезок с началом и концом, изображаемым нарисунке стрелкой. Произвольная точка пространства может считатьсянулевым вектором. Нулевой вектор не имеет конкретного направления,так как начало и конец совпадают, поэтому ему можно придать любоенаправление.

Длина(модуль) ненулевого вектора – это длина отрезка AB, котораяобозначается .Длина вектора обозначается .Нулевой вектор имеет длину равную нулю = 0.

Коллинеарныминазываются ненулевые векторы, лежащие на одной прямой или напараллельных прямых. На рисунках ниже изображены коллинеарные векторы,.

Нарисунке ниже также изображены коллинеарные векторы ,и .

Нулевойвектор коллинеарен любому вектору.

Сонаправленныминазываются коллинеарные ненулевые векторы, имеющие одно направление.Сонаправленные векторы обозначаются знаком ↑↑. Например,если вектор сонаправлен с вектором ,то используется запись ↑↑.На рисунках ниже изображены сонаправленные векторы.

Нулевойвектор сонаправлен с любым вектором.

Равныминазываются сонаправленные векторы равной длины. На рисунке нижеизображены равные векторы =.

Противоположнонаправленными называются два коллинеарных ненулевых вектора, имеющихпротивоположное направление. Противоположно направленные векторыобозначаются знаком ↑↓. Например, если вектор противоположно направлен вектору ,то используется запись ↑↓.На рисунках ниже изображены противоположно направленные векторы.

Противоположныминазываются два ненулевых противоположно направленных вектора равнойдлины. Вектор –является противоположным вектору

Противоположныевекторы противоположно направленные и равны по модулю. Если даныточки A и B,то следующие векторы противоположные

Равныепо модулю равные или противоположные, или нулевые векторы.

Многиефизические величины являются векторными величинами: сила, скорость,электрическое поле и т.д.

Еслине задана точка приложения (начала) вектора, то она выбираетсяпроизвольно.

Еслив точку O поместить начало вектора, тосчитается, что вектор отложен от точки O.Из любой точки можно отложить единственный вектор, равный данномувектору.

Источник: http://super-code.ru/vektor-1/vektor-1.html

3 Суммавекторов

Длясложения двух векторов по правилу треугольника проводится вектор 1,из конца этого вектора 1 проводится вектор 2, суммой данных вектора 1и вектора 2 является вектор 3, проведенный из начала вектора 1 кконцу вектора 2:

Дляпроизвольных точек A, Bи C можно написать сумму векторов:

+=

Еслидва вектора выходят из одной точки

тоих лучше складывать по правилу параллелограмма.

Длясложения двух векторов по правилу параллелограмма, складываемыевекторы 1 и 2 откладываются из одной точки, из концов этих векторов 1и 2 достраивается параллелограмм путем прикладывания к концу одноговектора начала другого. Вектор 3, образованный диагональюпараллелограмма, берущий начало от точки начала складываемыхвекторов, будет являться суммой векторов 1 и 2 (см. рисунок ниже).

Правилопараллелограмма содержит в себе два разных порядка сложения векторовпо правилу треугольника. Можно к концу вектора приложить начало вектора или к концу вектора приложить начало вектора .Результаты сложения в этих случаях и будут определяться по правилутреугольника.

Законысложения векторов:

1.Переместительный закон + = + .

2.Сочетательный закон (+ )+ = + (+ ).

Еслинеобходимо сложить несколько векторов, то векторы складываютсяпопарно или по правилу многоугольника: из конца вектора 1 проводитсявектор 2, из конца вектора 2 проводится вектор 3, из конца вектора 3проводится вектор 4, из конца вектора 4 проводится вектор 5 и т. д.Вектор, являющийся суммой нескольких векторов, проводится от началавектора 1 до конца последнего складываемого вектора.

Позаконам сложения векторов порядок сложения векторов не влияет нарезультирующий вектор, являющийся суммой нескольких векторов.

Источник: http://super-code.ru/vektor-1/vektor-1.html

Обобщения

Понятие вектора нормали можно обобщить на

  1. аффинные подпространства (обобщенные уровни) в евклидовых пространствах более высоких размерностей (математика) (особенно на гиперплоскостях ),
  2. Поверхности, гиперповерхности и подмногообразия в евклидовых пространствах более высоких размерностей,
  3. Поверхности, гиперповерхности и подмногообразия римановых многообразий ,
  4. Негладкие объекты, такие как выпуклые тела и спрямляемые множества .

Источник: http://deru.other.wiki/wiki/Normalenvektor

6 Произведениевектора на число

Произведениененулевого вектора на число k дает вектор

=k,

коллинеарныйвектору .

Например,вектор= 2дает вектор сонаправленный вектору и в 2 раза длиннее вектора .

Например,вектор= -4дает вектор противоположно направленный вектору и в 4 раза длиннее вектора .

Длинавектора :

||= |k|·||.

Длясоотношения векторов

=k

проанализируемвсе возможные варианты в зависимости от коэффициента k:

–если k > 0, товекторы и сонаправленные.

–если k = 0, товектор нулевой.

–если k < 0, товекторы и противоположно направленные.

–если |k| = 1, товекторы и равной длины.

–если k = 1, то и равные векторы.

–если k = -1, то и противоположные векторы.

–если |k| > 1,то длина вектора больше длины вектора .

–если k > 1, товекторы и сонаправленные и длина больше длины вектора .

–если k < -1,то векторы и противоположно направленные и длина больше длины вектора .

–если |k| < 1,то длина вектора меньше длины вектора .

–если 0 < k <1, то векторы и сонаправленные и длина меньше длины вектора .

–если -1 < k <0, то векторы и противоположно направленные и длина меньше длины вектора .

Произведениенулевого вектора на любое число дает нулевой вектор.

Задача.Дан вектор

Построитьвекторы 2,-3,0,5,-1,5.

Задача.Даны векторы и .

Построитьвекторы 3+2.

Решение

Векторы2– 2,-2– предлагается построить самостоятельно.

Законы,описывающие умножение вектора на число

1.Сочетательный закон (kn)=k(n)

2.Первый распределительный закон k(+)= k+k.

3.Второй распределительный закон (k + n)= k+n.

Дляколлинеарных векторов и ,если ≠ 0, существует единственное число k,позволяющее выразить вектор через :

=k.

Эточисло равно

Источник: http://super-code.ru/vektor-1/vektor-1.html

8 Разложениевектора по двум неколлинеарным векторам

Любойвектор единственным образом разлагается на плоскости по двум неколлинеарнымненулевым векторам и с единственными коэффициентами разложения xи y:

=x+ y

Любойвектор ,компланарный ненулевым неколлинеарным векторам и ,единственным образом разлагается по этим двум векторам и с единственными коэффициентами разложения xи y:

=x+ y

Разложимна плоскости заданный ненулевой вектор по данным ненулевым неколлинеарным векторам и :

Есливекторы ,и компланарные, то на плоскости проведем их из одной точки, построиввекторы равные заданным компланарным векторам

Изконца вектора проведем прямые, параллельные векторам и до пересечения с прямыми, проведенными через вектора и .Получим параллелограмм

Длинысторон параллелограмма получаются путем умножения длин векторов и на числа x и y,которые определяются путем деления длин сторон параллелограмма намодули (длины) соответствующих им векторов и .Получаем разложение вектора по заданным неколлинеарным векторам и :

=x+ y

Врешаемой задаче x ≈ 1,3, y≈ 1,9, поэтому разложение вектора по заданным неколлинеарным векторам и можно записать в виде

=1,3+ 1,9.

Разложимна плоскости заданный ненулевой вектор по данным ненулевым неколлинеарным векторам и :

Врешаемой задаче x ≈ 1,3, y≈ -1,9, поэтому разложение вектора по заданным неколлинеарным векторам и можно записать в виде

=1,3– 1,9.

Источник: http://super-code.ru/vektor-1/vektor-1.html

Перпендикулярные векторы в физике

В физике перпендикулярность некоторых векторов достаточно важна.

Вот несколько примеров:

  1. Если угол между вектором скорости тела и вектором силы, действующей на тело, будет прямым, то такая сила работу по перемещению тела совершать не будет.
  2. На проводник с током магнитное поле действует максимальной силой, когда вектор магнитной индукции и вектор тока в проводнике перпендикулярны.
  3. Когда угол между вращающей силой и, расстоянием между точкой приложения силы и осью вращения, будет прямым, вращательный момент будет максимальным.
  4. Между линейной скоростью точки колеса и расстоянием от этой точки до оси вращения, угол прямой (радиус и касательная перпендикулярны).
  5. На вращающееся тело действует центростремительная сила. Угол прямой между этой силой и линейной скоростью точки тела (радиус и касательная перпендикулярны).

Оценка статьи:

(голосов:

1

, средняя оценка:

5,00

из 5)

Загрузка…

Перпендикулярность векторов Ссылка на основную публикацию

Источник: http://formulki.ru/vektory/perpendikulyarnost-vektorov

веб ссылки

  • Эрик В. Вайсштейн : нормальный вектор . В: MathWorld (английский).

Источник: http://deru.other.wiki/wiki/Normalenvektor

11 Прямоугольнаясистема координат в пространстве

Втрехмерном пространстве прямоугольная система координат Oxyzзадается началом координат O ипересекающими в ней взаимно перпендикулярными координатными осями Ox,Oy и Oz свыбранными положительными направлениями, указанными стрелками, иединицей измерения отрезков. Если масштаб отрезков одинаковый по всемтрем осям, то такая система называется декартовой системой координат.

Координатаx называется абсциссой, y– ординатой, z – аппликатой. Координатыточки Mвзаданной системе координатзаписываютсяв скобках M (x;y; z).

Источник: http://super-code.ru/vektor-1/vektor-1.html

16 Уголмежду векторами

Уголмежду векторами – равен углу между лучами, проведенными изодной точки и сонаправленными с этими векторами. Обозначим угол междувекторами и буквой θ (тета) и два возможных варианта взаимного расположениявекторов изобразим на рисунках.

Уголмежду векторами может быть от 00 до 1800включительно

00≤θ≤ 1800.

Уголмежду сонаправленными векторами равен 00. Если один векторили оба нулевые, то угол между векторами, хотя бы один из которыхнулевой, равен 00. Угол между перпендикулярными векторамиравен 900. Угол между противоположно направленнымивекторами 1800.

Источник: http://super-code.ru/vektor-1/vektor-1.html

Взаимно перпендикулярный вектор — Большая Энциклопедия Нефти и Газа, статья, страница 1

Взаимно перпендикулярный вектор

Cтраница 1

Взаимно перпендикулярные векторы Е и Н напряженностей поля электромагнитной волны колеблются в одной фазе — они одновременно обращаются в нуль и одновременно достигают максимальных значений. Модули их связаны соотношением (30.12), которое справедливо для любой бегущей электромагнитной волны независимо от формы ее волновых поверхностей.  [1]

Взаимно перпендикулярные векторы Е и В в электромагнитной волне, распространяющейся в свободном пространстве, колеблются в одной фазе — они одновременно обращаются в нуль и одновременно достигают максимальных значений.  [2]

Направления двух взаимно перпендикулярных векторов плоскости называются главными направлениями аффинного преобразования, если образы этих векторов также взаимно перпендикулярны.  [3]

Таким образом, взаимно перпендикулярные векторы Е и Н колеблются в одной фазе — они одновременно обращаются в нуль и одновременно достигают максимальных значений. Соотношение (3.5), полученное для плоской волны, справедливо для любой бегущей электромагнитной волны независимо от формы ее волновых поверхностей.  [4]

Таким образом, взаимно перпендикулярные векторы Е и Н колеблются в одной фазе — они одновременно обращаются в нуль и одновременно достигают максимальных значений. Соотношение (3.5) справедливо также и для сферической волны.  [5]

Скалярное произведение двух взаимно перпендикулярных векторов равно нулю.  [6]

Скалярное произведение двух взаимно перпендикулярных векторов равно нулю.  [7]

Скалярное произведение двух взаимно перпендикулярных векторов равно нулю. Ясно, что ( а, Ь) ( Ь, а) и что скалярное произведение двух векторов равно произведению длины одного вектора на проекцию на него другого.  [8]

Система двух единичных взаимно перпендикулярных векторов называется прямоугольным базисом на плоскости.  [9]

Направления двух единичных взаимно перпендикулярных векторов называются перпендикулярными направлениями, а векторы, имеющие такие направления, называются взаимно перпендикулярными.  [10]

Скалярное произведение двух взаимно перпендикулярных векторов равно нулю.  [11]

J) и — q — единичные взаимно перпендикулярные векторы.  [12]

Покажите, что сумма и разность двух взаимно перпендикулярных векторов имеют одинаковую длину и также взаимно перпендикулярны.  [13]

Второе слагаемое (2.7) представляет собой скалярное произведение двух взаимно перпендикулярных векторов и поэтому равно нулю.  [14]

Страницы:      1    2    3

Скалярное произведение в координатах

На прошлых занятиях к уже известным действиям над векторами, а именно сложению, вычитанию и умножению вектора на число, мы добавили скалярное умножение векторов.

Мы говорили, что результатом первых трёх действий является некоторый вектор, а вот результатом скалярного умножения векторов — число.

Причём скалярное произведение равно 0, если хотя бы один из векторов нулевой. В случае, когда оба вектора ненулевые, скалярное произведение может принимать положительное значение, если угол между векторами острый, отрицательное значение, если угол между векторами тупой. Скалярное произведение ненулевых векторов равно нулю тогда и только тогда, когда эти векторы перпендикулярны.

Вы знаете, что каждый вектор имеет свои координаты. В связи с этим ранее нами были получены правила, позволяющие выражать координаты вектора, представленного в виде алгебраической суммы данных векторов с известными координатами.

Так каждая координата вектора суммы равна сумме соответствующих координат данных векторов.

,

Каждая координата вектора разности двух векторов равна разности соответствующих координат данных векторов.

,

И каждая координата произведения вектора на число равна произведению соответствующих координат данного вектора на это число.

Понятно, что и скалярное произведение векторов можно выразить некоторым образом через координаты данных векторов.

Теорема. В прямоугольной системе координат скалярное произведение векторов  и  выражается формулой:

Доказательство.

,

,

,

, , то

 

 

,

 

Что и требовалось доказать.

Скалярное произведение векторов равно сумме произведений их соответствующих координат.

Выполним задание, где применим эту формулу.

Задача. Найти скалярное произведение векторов , , , если , , .

Решение.

Получается, что, зная координаты векторов, мы можем выяснить, перпендикулярны они или нет.

И в связи с изученной сегодня формулой, можно записать следствие из теоремы.

Следствие 1. Ненулевые векторы  и  перпендикулярны тогда и только тогда, когда .

Какие из данных векторов являются перпендикулярными для вектора ?

                                                   

Для этого сумма произведений соответствующих координат векторов должна быть равна нулю.

Составим такие выражения для вектора с каждым из векторов ,   и .

Проверим пары векторов.

 

 

 

 

Мы получили, что только два вектора,  и , являются перпендикулярными вектору .

А теперь рассмотрим ещё одно следствие из теоремы.

Следствие 2. Косинус угла  между ненулевыми векторами  и  выражается формулой: .

 

Действительно, из формулы скалярного произведения

.

Найти косинусы углов между векторами , , .  Если , , , , , .

Запишем формулу косинуса угла между векторами

Так мы рассмотрели примеры вычисления косинуса угла между векторами по их координатам.

А теперь выполним такое задание.

Задача. Определить, при каких значениях переменной  .

,

,

,

,

Решение.

По следствию, записанному сегодня, можно записать, что для того, чтобы векторы  были перпендикулярны, сумма произведений их координат должна равняться нулю.

 

  

 

 

 

 

 

 

 

 

 

В каждом из этих пунктов мы определили значение переменной x, при котором векторы будут перпендикулярны.

Задача. Доказать, что треугольник с вершинами ,  и  тупоугольный и найти косинус тупого угла.

Решение.

Нужно доказать, что треугольник с вершинами А, B и C тупоугольный и найти косинус тупого угла.

В нашем треугольнике ABC три угла. Чтобы доказать, что он тупоугольный достаточно найти косинус каждого угла и проанализировать его величину.

Если косинус одного из углов окажется меньшим нуля, то тем самым мы докажем, что данный угол тупой, а треугольник — тупоугольный.

Мы умеем находить косинус угла между векторами по их координатам.

Чтобы воспользоваться изученной формулой косинуса угла между векторами, нужно знать их координаты.

Найдём их как разности соответствующих координат конца и начала вектора.

 

 острый

 острый

 тупой

Тем самым мы доказали, что треугольник ABC — тупоугольный, и нашли косинус тупого угла.

Что и требовалось доказать.

Ответ: .

Подведём итоги урока.

Сегодня мы с вами получили формулу скалярного произведения векторов в координатах.

А также сформулировали следствия из данной теоремы.

Первое из них гласит, что ненулевые векторы перпендикулярны тогда и только тогда, когда сумма произведений их соответствующих координат равна нулю.

Второе следствие позволяет находить косинус угла между векторами через их координаты.

Перпендикулярность векторов, условие перпендикулярности, примеры

Мы можем выяснить, будут ли два каких-либо вектора взаимно перпендикулярными. Для этого нужно воспользоваться координатами векторов и некоторыми приемами, описанными в данной статье. Информация о перпендикулярности будет полезной для решения некоторых задач физики и математики.

Координаты вектора на плоскости, равного по модулю и перпендикулярного данному

Пусть на плоскости заданы координаты какого-либо вектора. Из этих координат получим координаты двух дополнительных векторов, перпендикулярных первоначальному вектору. Все три вектора будут иметь равные длины и располагаться в плоскости xOy.

Алгоритм получения координат перпендикулярных векторов

Вектор на плоскости xOy, перпендикулярный данному вектору получают так:

  1. Поменять местами координатные числа «x» и «y».
  2. Заменить знак у одной из координат на противоположный.

Графический пример

Рассмотрим небольшой графический пример (рис. 1).

Рис. 1. На рисунке векторы, обозначенные черным цветом, перпендикулярны вектору, обозначенному красным цветом

На плоскости проведены три вектора: один красный и два черных и, отмечены их координаты. Рассмотрим подробнее координаты двух векторов: \(\vec{a}\) и \(\vec{b}\).

\[ \vec{a} = \left\{ 4 ; 3 \right\} \]

\[ \vec{b} = \left\{ -3 ; 4 \right\} \]

Из рисунка видно, что векторы \(\vec{a}\) и \(\vec{b}\) перпендикулярны: \( \vec{a} \perp \vec{b} \).

Вектор \( -\vec{b} = \left\{ 3 ; -4 \right\} \), также будет перпендикулярным вектору \( \vec{a} \): \( \vec{a} \perp \vec{(-b)} \)

Векторы, изображенные черным цветом, перпендикулярны красному вектору.

Длины векторов \( \vec{a} \), \( \vec{b} \) и \( \vec{(-b)} \) равны.

 Условие перпендикулярности векторов

Взаимную перпендикулярность двух векторов можно проверить, вычислив их скалярное произведение. Этот способ проверки можно применять для векторов, расположенных как на плоскости, так и в трехмерном пространстве.

Векторы будут перпендикулярными, когда их скалярное произведение равно нулю.

Пусть, известны координаты двух векторов и пусть каждый вектор имеет ненулевую длину.

\[ \large \boxed { \begin{cases} \vec{a} = \left\{ a_{x} ; a_{y} ; a_{z} \right\} \\ \vec{b} = \left\{ b_{x} ; b_{y} ; b_{z} \right\} \\ |\vec{a}| \ne 0  \\ |\vec{b}| \ne 0 \end{cases}}\]

Запишем условие перпендикулярности векторов.

Для двумерного случая:

\[ \large \boxed { a_{x} \cdot b_{x} + a_{y} \cdot b_{y} = 0 }\]

Для трехмерного случая:

\[ \large \boxed { a_{x} \cdot b_{x} + a_{y} \cdot b_{y} + a_{z} \cdot b_{z} = 0 }\]

Пользуясь любой из этих формул, можно определить одну неизвестную координату вектора.

При этом, должны быть известными остальные координаты этого вектора и все координаты второго вектора.

Примечание:

Есть такое правило: Количество неизвестных должно равняться количеству уравнений.

Чтобы однозначно определить значение неизвестной, в уравнение должна входить только одна неизвестная. Остальные величины должны быть известными.

Перпендикулярные векторы в физике

В физике перпендикулярность некоторых векторов достаточно важна.

Вот несколько примеров:

  1. Если угол между вектором скорости тела и вектором силы, действующей на тело, будет прямым, то такая сила работу по перемещению тела совершать не будет.
  2. На проводник с током магнитное поле действует максимальной силой, когда вектор магнитной индукции и вектор тока в проводнике перпендикулярны.
  3. Когда угол между вращающей силой и, расстоянием между точкой приложения силы и осью вращения, будет прямым, вращательный момент будет максимальным.
  4. Между линейной скоростью точки колеса и расстоянием от этой точки до оси вращения, угол прямой (радиус и касательная перпендикулярны).
  5. На вращающееся тело действует центростремительная сила. Угол прямой между этой силой и линейной скоростью точки тела (радиус и касательная перпендикулярны).

 

Может ли скалярное произведение быть отрицательным

Вектор можно не только умножать на число, но и перемножать вектора между собой. Такое действие носит название скалярного произведения векторов.

Если сложение или вычитание векторов является, в общем-то, достаточно понятным и логичным действием, которое просто отображается графически, то с пониманием скалярного произведения векторов не всё так просто. Гораздо легче запомнить формулу скалярного умножения двух векторов, благо, она куда проще для запоминания, нежели для понимания.

  • скалярное произведение перпендикулярных векторов равно нулю, т.к., cos(90°)=0;
  • если хотя бы один из перемножаемых векторов является нулевым, то скалярное произведение также будет равно нулю;
  • если ни один из перемножаемых векторов не является нулевым, а скалярное произведение векторов равно нулю, то угол между такими векторами будет прямым;
  • если угол между векторами тупой, то скалярное произведение таких векторов будет отрицательным числом;
  • если угол между векторами острый, то скалярное произведение таких векторов будет положительным числом;
  • произведение вектора на самого себя будет равно квадрату модуля этого вектора.

Все же попробуем разъяснить суть умножения двух векторов.

Начнем с самого простого случая, когда два перемножаемых между собой вектора являются сонаправленными, т. е., угол между такими векторами будет равен 0°.

Таким образом, произведение сонаправленных векторов будет равно произведению их модулей, т.е., умножается длина одного вектора на длину второго вектора.

ВАЖНО : Обратите внимание, что результатом скалярного произведения векторов будет число, а не вектор!

А как быть, если перемножаемые между собой вектора расположены под некоторым углом друг к другу?

В этом случае в дело вступает косинус величины угла, под которым располагаются друг к другу перемножаемые вектора.

Возьмём ещё один частный случай, когда два вектора расположены перпендикулярно друг к другу, т.е. образуют угол в 90°.

Как видим, произведение двух перпендикулярых векторов равно 0.

Почему так происходит?

Нюанс перемножения двух векторов, расположенных друг к другу под некоторым углом, заключается в том, что перемножается модуль первого вектора на модуль проекции на этот вектор второго вектора. В случае перпендикулярности проекция второго вектора на первый будет являться точкой или нулевым вектором, а скалярное произведение вектора на нулевой вектор равно нулю (см. выше).

Возьмём третий возможный случай, когда угол между векторами острый, т.е., лежит в пределах от 0° до 90°.

Как известно, косинус углов в этом диапазоне будет числом положительным — меньше 1, но больше 0. Поэтому, скалярное произведение двух не нулевых векторов, расположенных друг к другу под острым углом, всегда будет числом положительным (не следует забывать, что модуль вектора не может быть числом отрицательным, поскольку это расстояние между двумя точками).

В случае, если угол между перемножаемыми векторами будет тупой, т.е., больше 90°, но меньше 180°, в таком случае произведение таких векторов будет числом отрицательным, поскольку в этом диапазоне косинус колеблется от 0 до -1.

И последний частный возможный случай — перемножаемые вектора являются противопложно направленными, т.е., образуют развернутый угол (180°). В таком случае, поскольку cos(180°)=-1, скалярное произведение противоположно направленных векторов будет равно отрицательному произведению их модулей.

Скалярное произведение векторов, заданных координатами

Формула для вычисления длины вектора а с координатами :

Свойства скалярного произведения векторов:

  • переместительный закон: ab = ba;
  • распределительный закон: (a+b)c = ac+bc;
  • сочетательный закон: (ka)b = k(ab), где k — любое число;
  • отрицательный квадрат не нулевого вектора будет положительным числом: а -2 >0.

Если вам понравился сайт, будем благодарны за его популяризацию 🙂 Расскажите о нас друзьям на форуме, в блоге, сообществе. Это наша кнопочка:

Код кнопки:
Политика конфиденциальности Об авторе

Скалярное произведение векторов называют число, равное произведению дин этих векторов на косинус угла между ними. = a → · n p a → b → = b → · n p b → a → показывает, что n p b → a → — это числовая проекция a → на b → , n p a → a → — проекция b → на a → соостветсвенно.

Сформулируем определение произведения для двух векторов:

Скалярное произведение двух векторов a → на b → называют произведение длины вектора a → на проекцию b → на направление a → или произведение длины b → на проекцию a → соответственно.

Скалярное произведение в координатах

Вычисление скалярного произведения можно производить через координаты векторов в заданной плоскости или в пространстве.

Скаларное произведение двух векторов на плоскости, в трехмерном простарнстве называют сумму координат заданных векторов a → и b → .

При вычислении на плоскости скаларного произведения заданных векторов a → = ( a x , a y ) , b → = ( b x , b y ) в декартовой системе используют:

a → , b → = a x · b x + a y · b y ,

для трехмерного пространства применимо выражение:

a → , b → = a x · b x + a y · b y + a z · b z . ) = = a x · b x + a y · b y + a z · b z

– соответственно для векторов трехмерного пространства.

Скалярное произведение векторов с координатами говорит о том, что скалярный квадрат вектора равен сумме квадратов его координат в пространстве и на плоскости соответственно. a → = ( a x , a y , a z ) , b → = ( b x , b y , b z ) и ( a → , a → ) = a x 2 + a y 2 .

Скалярное произведение и его свойства

Существуют свойства скалярного произведения, которые применимы для a → , b → и c → :

  1. коммутативность ( a → , b → ) = ( b → , a → ) ;
  2. дистрибутивность ( a → + b → , c → ) = ( a → , c → ) + ( b → , c → ) , ( a → + b → , c → ) = ( a → , b → ) + ( a → , c → ) ;
  3. сочетательное свойство ( λ · a → , b → ) = λ · ( a → , b → ) , ( a → , λ · b → ) = λ · ( a → , b → ) , λ — любое число;
  4. скалярный квадрат всегда больше нуля ( a → , a → ) ≥ 0 , где ( a → , a → ) = 0 в том случае, когда a → нулевой.

Пример 1

Свойства объяснимы благодаря определению скалярного произведения на плоскости и свойствам при сложении и умножении действительных чисел.

Доказать свойство коммутативности ( a → , b → ) = ( b → , a → ) . Из определения имеем, что ( a → , b → ) = a y · b y + a y · b y и ( b → , a → ) = b x · a x + b y · a y .

По свойству коммутативности равенства a x · b x = b x · a x и a y · b y = b y · a y верны, значит a x · b x + a y · b y = b x · a x + b y · a y .

Отсюда следует, что ( a → , b → ) = ( b → , a → ) . Что и требовалось доказать.

Дистрибутивность справедлива для любых чисел:

( a ( 1 ) → + a ( 2 ) → + . . . + a ( n ) → , b → ) = ( a ( 1 ) → , b → ) + ( a ( 2 ) → , b → ) + . . . + ( a ( n ) → , b → )

и ( a → , b ( 1 ) → + b ( 2 ) → + . . . + b ( n ) → ) = ( a → , b ( 1 ) → ) + ( a → , b ( 2 ) → ) + . . . + ( a → , b → ( n ) ) ,

( a ( 1 ) → + a ( 2 ) → + . . . + a ( n ) → , b ( 1 ) → + b ( 2 ) → + . . . + b ( m ) → ) = = ( a ( 1 ) → , b ( 1 ) → ) + ( a ( 1 ) → , b ( 2 ) → ) + . . . + ( a ( 1 ) → , b ( m ) → ) + + ( a ( 2 ) → , b ( 1 ) → ) + ( a ( 2 ) → , b ( 2 ) → ) + . . . + ( a ( 2 ) → , b ( m ) → ) + . ) = 3 · 7 · cos 60 ° = 3 · 7 · 1 2 = 21 2

Ответ: ( a → , b → ) = 21 2 .

Заданны векторы a → = ( 1 , — 1 , 2 — 3 ) , b → = ( 0 , 2 , 2 + 3 ) . Чему равно скалярной произведение.

В данном примере рассматривается формула вычисления по координатам, так как они заданы в условии задачи:

( a → , b → ) = a x · b x + a y · b y + a z · b z = = 1 · 0 + ( — 1 ) · 2 + ( 2 + 3 ) · ( 2 + 3 ) = = 0 — 2 + ( 2 — 9 ) = — 9

Ответ: ( a → , b → ) = — 9

Найти скалярное произведение A B → и A C → . На координатной плоскости заданы точки A ( 1 , — 3 ) , B ( 5 , 4 ) , C ( 1 , 1 ) .

Для начала вычисляются координаты векторов, так как по условию даны координаты точек:

A B → = ( 5 — 1 , 4 — ( — 3 ) ) = ( 4 , 7 ) A C → = ( 1 — 1 , 1 — ( — 3 ) ) = ( 0 , 4 )

Подставив в формулу с использованием координат, получим:

( A B → , A C → ) = 4 · 0 + 7 · 4 = 0 + 28 = 28 .

Ответ: ( A B → , A C → ) = 28 .

Заданы векторы a → = 7 · m → + 3 · n → и b → = 5 · m → + 8 · n → , найти их произведение. m → равен 3 и n → равен 2 единицам, они перпендикулярные.

( a → , b → ) = ( 7 · m → + 3 · n → , 5 · m → + 8 · n → ) . Применив свойство дистрибутивности, получим:

( 7 · m → + 3 · n → , 5 · m → + 8 · n → ) = = ( 7 · m → , 5 · m → ) + ( 7 · m → , 8 · n → ) + ( 3 · n → , 5 · m → ) + ( 3 · n → , 8 · n → )

Выносим коэффициент за знак произведения и получим:

( 7 · m → , 5 · m → ) + ( 7 · m → , 8 · n → ) + ( 3 · n → , 5 · m → ) + ( 3 · n → , 8 · n → ) = = 7 · 5 · ( m → , m → ) + 7 · 8 · ( m → , n → ) + 3 · 5 · ( n → , m → ) + 3 · 8 · ( n → , n → ) = = 35 · ( m → , m → ) + 56 · ( m → , n → ) + 15 · ( n → , m → ) + 24 · ( n → , n → )

По свойству коммутативности преобразуем:

35 · ( m → , m → ) + 56 · ( m → , n → ) + 15 · ( n → , m → ) + 24 · ( n → , n → ) = = 35 · ( m → , m → ) + 56 · ( m → , n → ) + 15 · ( m → , n → ) + 24 · ( n → , n → ) = = 35 · ( m → , m → ) + 71 · ( m → , n → ) + 24 · ( n → , n → )

В итоге получим:

( a → , b → ) = 35 · ( m → , m → ) + 71 · ( m → , n → ) + 24 · ( n → , n → ) . ) + 24 · n → 2 = = 35 · 3 2 + 71 · 3 · 2 · cos π 2 + 24 · 2 2 = 411 .

Ответ: ( a → , b → ) = 411

Если имеется числовая проекция.

Найти скалярное произведение a → и b → . Вектор a → имеет координаты a → = ( 9 , 3 , — 3 ) , проекция b → с координатами ( — 3 , — 1 , 1 ) .

По условию векторы a → и проекция b → противоположно направленные, потому что a → = — 1 3 · n p a → b → → , значит проекция b → соответствует длине n p a → b → → , при чем со знаком «-»:

n p a → b → → = — n p a → b → → = — ( — 3 ) 2 + ( — 1 ) 2 + 1 2 = — 11 ,

Подставив в формулу, получим выражение:

( a → , b → ) = a → · n p a → b → → = 9 2 + 3 2 + ( — 3 ) 2 · ( — 11 ) = — 33 .

Ответ: ( a → , b → ) = — 33 .

Задачи при известном скалярном произведении, где необходимо отыскать длину вектора или числовую проекцию.

Какое значение должна принять λ при заданном скалярном произведении a → = ( 1 , 0 , λ + 1 ) и b → = ( λ , 1 , λ ) будет равным -1.

Из формулы видно, что необходимо найти сумму произведений координат:

( a → , b → ) = 1 · λ + 0 · 1 + ( λ + 1 ) · λ = λ 2 + 2 · λ . ) = 5 · 3 · cos ( 45 ° ) = 15 2 2 .

Ответ: A = 15 2 2 .

Материальная точка, перемещаясь из M ( 2 , — 1 , — 3 ) в N ( 5 , 3 λ — 2 , 4 ) под силой F → = ( 3 , 1 , 2 ) , совершила работа равную 13 Дж. Вычислить длину перемещения.

При заданных координатах вектора M N → имеем M N → = ( 5 — 2 , 3 λ — 2 — ( — 1 ) , 4 — ( — 3 ) ) = ( 3 , 3 λ — 1 , 7 ) .

По формуле нахождения работы с векторами F → = ( 3 , 1 , 2 ) и M N → = ( 3 , 3 λ — 1 , 7 ) получим A = ( F ⇒ , M N → ) = 3 · 3 + 1 · ( 3 λ — 1 ) + 2 · 7 = 22 + 3 λ .

По условию дано, что A = 13 Д ж , значит 22 + 3 λ = 13 . Отсюда следует λ = — 3 , значит и M N → = ( 3 , 3 λ — 1 , 7 ) = ( 3 , — 10 , 7 ) .

Чтобы найти длину перемещения M N → , применим формулу и подставим значения:

Устанавливая рекомендуемое программное обеспечение вы соглашаетесь
с лицензионным соглашением Яндекс.Браузера и настольного ПО Яндекса .

Описание презентации по отдельным слайдам:

Угол между векторами a О

№ 1039 Диагонали квадрата пересекаются в точке О. Найдите углы между векторами. 450 А В С D О 900 900 1800 900 1350 00

Сумма векторов – вектор. Разность векторов – вектор. Произведение вектора на число – вектор. Скалярное произведение векторов – число.

Скалярным произведением двух векторов называется произведение их длин на косинус угла между ними.

№ 1041 Вычислите скалярное произведение векторов и , если ׀ ׀=2, ׀ b ׀=3, а угол межу ними равен: а) 45º; б) 90º; в) 135º. cos( ) a a b = б) 2·3·cos90º = 2·3·0 = 0 в) a a b а)

Скалярное произведение ненулевых векторов положительно тогда и только тогда , когда угол между векторами острый. Скалярное произведение ненулевых векторов отрицательно тогда и только тогда , когда угол между векторами тупой.

Устанавливая рекомендуемое программное обеспечение вы соглашаетесь
с лицензионным соглашением Яндекс.Браузера и настольного ПО Яндекса .

  • Прудченко Ирина ВячеславовнаНаписать 521 13.12.2018

Номер материала: ДБ-300878

Устанавливая рекомендуемое программное обеспечение вы соглашаетесь
с лицензионным соглашением Яндекс.Браузера и настольного ПО Яндекса .

Не нашли то что искали?

Вам будут интересны эти курсы:

Все материалы, размещенные на сайте, созданы авторами сайта либо размещены пользователями сайта и представлены на сайте исключительно для ознакомления. Авторские права на материалы принадлежат их законным авторам. Частичное или полное копирование материалов сайта без письменного разрешения администрации сайта запрещено! Мнение редакции может не совпадать с точкой зрения авторов.

Ответственность за разрешение любых спорных моментов, касающихся самих материалов и их содержания, берут на себя пользователи, разместившие материал на сайте. Однако редакция сайта готова оказать всяческую поддержку в решении любых вопросов связанных с работой и содержанием сайта. Если Вы заметили, что на данном сайте незаконно используются материалы, сообщите об этом администрации сайта через форму обратной связи.

Как найти вектор перпендикулярный вектору

ФОРМУЛА

Чтобы вектор \(\ \overline{a}\) был перпендикулярен вектору \(\ \overline{b}\) , необходимо, чтобы его скалярное произведение было равно нулю, т.е.

\(\ (\overline{a}, \overline{b})=0 \)

Если векторы задаются на плоскости своими координатами \(\ \overline{a}=\left(a_{x} ; a_{y}\right) \) и \(\ \overline{b}=\left(b_{x} ; b_{y}\right) \), то условие их перпендикулярности принимает вид:

\(\ (\overline{a}, \overline{b})=a_{x} \cdot b_{x}+a_{y} \cdot b_{y}=0 \)

Если векторы заданы в пространстве и имеют координаты \(\ \overline{a}=\left(a_{x} ; a_{y} ; a_{z}\right) \) и \(\ \overline{b}=\left(b_{x} ; b_{y} ; b_{z}\right) \), то перпендикулярное условие записывается в виде:

\(\ (\overline{a}, \overline{b})=a_{x} \cdot b_{x}+a_{y} \cdot b_{y}+a_{z} \cdot b_{z}=0 \)

ПРИМЕРЫ РАСПОЛОЖЕНИЯ ПЕРПЕНДИКУЛЯРНОГО ВЕКТОРА

ПРИМЕР

  • Задание: Даны два вектора \(\ \overline{a}=(2 ;-1) \) и \(\ \overline{b}=(-3 ; m) \) . При каком значении \(\ m \) эти векторы будут перпендикулярны?
  • Решение: Чтобы векторы \(\ \overline{a} \) и \(\ \overline{b} \) были перпендикулярны, необходимо, чтобы их скалярное произведение было равно нулю, то есть условие выполняется:

    \(\ (\overline{a}, \overline{b})=a_{x} \cdot b_{x}+a_{y} \cdot b_{y}=0 \)

    Подставляем координаты указанных векторов в это выражение и из полученного равенства находим \(\ m \):

    \(\ 2 \cdot(-3)+(-1) \cdot m=0 \)

    \(\ -6-m=0 \)

    \(\ m=-6 \)

  • Ответ: Векторы \(\ \overline{a} \) и \(\ \overline{b} \) будут перпендикулярны \(\ m=-6 \)

    ПРИМЕР

  • Задание: Два вектора \(\ \overline{a}=(3 ;-2 ; m) \) и \(\ \overline{b}=(-1 ; m ; 1) \) даны. При каком значении \(\ m \) эти векторы будут перпендикулярны?
  • Решение: \(\ \overline{a}\) и \(\ \overline{b}\) будут перпендикулярны, когда их скалярное произведение равно нулю. А поскольку векторы заданы в пространстве, условие должно быть выполнено:

    \(\ (\overline{a}, \overline{b})=a_{x} \cdot b_{x}+a_{y} \cdot b_{y}+a_{z} \cdot b_{z}=0 \)

    Подставив в него указанные координаты векторов, мы получим:

    \(\ 3 \cdot(-1)+(-2) \cdot m+m \cdot 1=0 \)

    \(\ 3-2 \cdot m+m=0 \)

    Из полученного уравнения находим \(\ m=-6 \):

    \(\ 3-m=0 \Rightarrow m=-3 \)

  • Ответ: Векторы \(\ \overline{a}\) и \(\ \overline{b}\) будут перпендикулярны \(\ m=-3 \)
  • 2 $.

    Сначала заметьте, что если один вектор лежит вдоль оси $ x $ $ u = x \ hat {i} $, а другой $ v = y \ hat {j} $ лежит вдоль оси $ y $, то их скалярное произведение равно нуль.

    Затем возьмем произвольную пару перпендикулярных векторов $ u, v $. Если мы можем повернуть их обоих так, чтобы они оба лежали вдоль осей и , скалярное произведение инвариантно при этом повороте, тогда мы получим результат.

    То есть пусть $ R $ будет линейным преобразованием или матрицей, такой что $ Ru $ и $ Rv $ лежат вдоль осей.Тогда, если это так, что $$ Ru \ cdot Rv = u \ cdot v \ \ \ — (\ star) $$, тогда мы закончили, потому что $ Ru \ cdot Rv = 0 $ и, следовательно, $ u \ cdot v = 0 $.

    Помните, что по нашему первому наблюдению $ Ru \ cdot Rv = 0 $.

    Это интуиция. Есть смысл?

    Теперь оказывается, что интуиция подтверждается, потому что ($ \ star $) верно. Далее следует некоторая алгебра, демонстрирующая ($ \ star $). Он занимает больше места, чем идея, что прискорбно, поскольку идея является более важной вещью.


    Поворот $ R $ для некоторого $ \ theta $ можно записать как

    $$ R = \ begin {pmatrix} \ cos \ theta \ \ sin \ theta \\ — \ sin \ theta \ \ cos \ theta \ end {pmatrix} $$

    Запись $ u $ и $ v $ сейчас как вектор-столбец с $ u = {u_x \ choose u_y} $ и $ v = {v_x \ choose v_y} $, затем

    $$ Ru = \ begin {pmatrix} \ cos \ theta \ \ sin \ theta \\ — \ sin \ theta \ \ cos \ theta \ end {pmatrix} \ begin {pmatrix} u_x \\ u_y \ end {pmatrix} = \ begin {pmatrix} u_x \ cos \ theta + u_y \ sin \ theta \\ -u_x \ sin \ theta + u_y \ cos \ theta \ end {pmatrix} $$

    Аналогично

    $$ Rv = \ begin {pmatrix} \ cos \ theta \ \ sin \ theta \\ — \ sin \ theta \ \ cos \ theta \ end {pmatrix} \ begin {pmatrix} v_x \\ v_y \ end {pmatrix} = \ begin {pmatrix} v_x \ cos \ theta + v_y \ sin \ theta \\ -v_x \ sin \ theta + v_y \ cos \ theta \ end {pmatrix} $$

    Отсюда

    $$ Ru \ cdot Rv = \ begin {pmatrix} u_x \ cos \ theta + u_y \ sin \ theta \\ -u_x \ sin \ theta + u_y \ cos \ theta \ end {pmatrix} \ cdot \ begin {pmatrix} v_x \ cos \ theta + v_y \ sin \ theta \\ -v_x \ sin \ theta + v_y \ cos \ theta \ end {pmatrix} $$

    $$ = (u_x \ cos \ theta + u_y \ sin \ theta) (v_x \ cos \ theta + v_y \ sin \ theta) + (-u_x \ sin \ theta + u_y \ cos \ theta) (- v_x \ sin \ theta + v_y \ cos \ theta) $$

    $$ = u_xv_x (\ cos ^ 2 \ theta + \ sin ^ 2 \ theta) + u_xv_y (\ sin \ theta \ cos \ theta — \ sin \ theta \ cos \ theta) + u_yv_x (\ sin \ theta \ cos \ theta — \ sin \ theta \ cos \ theta) + u_yv_y (\ cos ^ 2 \ theta + \ sin ^ 2 \ theta) $$

    $$ = u_xv_x + u_yv_y $$

    $$ = u \ cdot v $$

    То есть $ Ru \ cdot Rv = u \ cdot v $, как и нужно.{3} \) Выдано:

    \ [\ textbf {v} \ times \ textbf {w} = (v_ {2} w_ {3} — v_ {3} w_ {2}, v_ {3} w_ {1} — v_ {1} w_ { 3}, v_ {1} w_ {2} — v_ {2} w_ {1}) \]

    Пример 1.7

    Найдите \ (\ textbf {i} \ times \ textbf {j} \).

    Решение

    Поскольку \ (\ textbf {i} = (1,0,0) \) и \ (\ textbf {j} = (0,1,0) \), то
    \ begin {align *}
    \ textbf { i} \ times \ textbf {j} & = ((0) (0) — (0) (1), (0) (0) — (1) (0), (1) (1) — (0) (0)) \\ [4pt]
    & = (0,0,1) \\ [4pt]
    & = \ textbf {k}
    \ end {align *}

    Аналогичным образом можно показать, что \ (\ textbf {j} \ times \ textbf {k} = \ textbf {i} \) и \ (\ textbf {k} \ times \ textbf {i} = \ textbf {j} \).

    Рисунок 1.4.1

    В приведенном выше примере перекрестное произведение данных векторов было перпендикулярно обоим этим векторам. Оказывается, так будет всегда.

    Теорема 1.11

    Если векторное произведение \ (\ textbf {v} \ times \ textbf {w} \) двух ненулевых векторов \ (\ textbf {v} \) и \ (\ textbf {w} \) также является ненулевым вектором, тогда он перпендикулярен как \ (\ textbf {v} \), так и \ (\ textbf {w} \).

    Проба

    Мы покажем, что \ ((\ textbf {v} \ times \ textbf {w}) \ cdot \ textbf {v} = 0 \):
    \ begin {align *}
    (\ textbf {v} \ times \ textbf {w}) \ cdot \ textbf {v} & = (v_ {2} w_ {3} — v_ {3} w_ {2}, v_ {3} w_ {1} — v_ {1} w_ {3} , v_ {1} w_ {2} — v_ {2} w_ {1}) \ cdot (v_ {1}, v_ {2}, v_ {3}) \\ [4pt]
    & = v_ {2} w_ {3} v_ {1} — v_ {3} w_ {2} v_ {1} +
    v_ {3} w_ {1} v_ {2} — v_ {1} w_ {3} v_ {2} + v_ { 1} w_ {2} v_ {3} —
    v_ {2} w_ {1} v_ {3} \\ [4pt]
    & = v_ {1} v_ {2} w_ {3} — v_ {1} v_ {2} w_ {3} + w_ {1} v_ {2} v_ {3} —
    w_ {1} v_ {2} v_ {3} + v_ {1} w_ {2} v_ {3} — v_ { 1} w_ {2} v_ {3} \\ [4pt]
    & = 0 \ text {, после перестановки терминов.}
    \ end {align *}
    \ (\ следовательно \ textbf {v} \ times \ textbf {w} \ perp \ textbf {v} \) по следствию 1.7. Доказательство того, что \ (\ textbf {v} \ times \ textbf {w} \ perp \ textbf {w} \) аналогично.

    \ (\ квадрат \)

    Как следствие приведенной выше теоремы и теоремы 1.9, имеем:

    Следствие 1.12

    Если векторное произведение \ (\ textbf {v} \ times \ textbf {w} \) двух ненулевых векторов \ (\ textbf {v} \) и \ (\ textbf {w} \) также является ненулевым вектором, тогда он перпендикулярен промежутку между \ (\ textbf {v} \) и \ (\ textbf {w} \).{3} \) является плоскостью \ (P \), поэтому приведенное выше следствие показывает, что \ (\ textbf {v} \ times \ textbf {w} \) перпендикулярно этой плоскости. Как показано на рисунке 1.4.2, есть два возможных направления для \ (\ textbf {v} \ times \ textbf {w} \), одно противоположное другому. Оказывается (см. Приложение B), направление \ (\ textbf {v} \ times \ textbf {w} \) задается \ (\ textit {правило правой руки} \), то есть векторами \ (\ textbf {v}, \ textbf {w} \), \ (\ textbf {v} \ times \ textbf {w} \) образуют правостороннюю систему. Напомним из раздела 1.{3} \), затем

    \ [\ norm {\ textbf {v} \ times \ textbf {w}} = \ norm {\ textbf {v}} \, \ norm {\ textbf {w}} \, \ sin \ theta \]

    Может показаться странным возиться с приведенной выше формулой, когда величина векторного произведения может быть вычислена напрямую, как и для любого другого вектора. Формула более полезна для ее применения в геометрии, как в следующем примере.

    Пример 1.8

    Пусть \ (\ треугольник PQR \) и \ (PQRS \) будут треугольником и параллелограммом соответственно, как показано на рисунке 1.{3} \). Пусть \ (\ theta \) будет углом между \ (\ textbf {v} \) и \ (\ textbf {w} \). Площадь \ (A_ {PQR} \) \ (\ треугольник PQR \) равна \ (\ frac {1} {2} bh \), где \ (b \) — основание треугольника, а \ (h \ ) — высота. Итак, мы видим, что

    \ [\ nonumber b = \ norm {\ textbf {v}} \ text {and} h = \ norm {\ textbf {w}} \, \ sin \ theta \]

    \ [\ nonumber \ begin {align} A_ {PQR} & = \ frac {1} {2} \, \ norm {\ textbf {v}} \, \ norm {\ textbf {w}} \, \ sin \ theta \\ [4pt] \ nonumber & = \ frac {1} {2} \, \ norm {\ textbf {v} \ times \ textbf {w}} \\ [4pt] \ end {align} \]

    Итак, поскольку площадь \ (A_ {PQRS} \) параллелограмма \ (PQRS \) в два раза больше площади треугольника \ (\ Triangle PQR \), то

    \ [\ nonumber A_ {PQRS} = \ norm {\ textbf {v}} \, \ norm {\ textbf {w}} \, \ sin \ theta \]

    Согласно обсуждению в Примере 1.{3} \)):
    \ [\ nonumber A = \ norm {\ textbf {v} \ times \ textbf {w}} \]

    На первый взгляд может показаться, что, поскольку формулы, полученные в примере 1.8, относились только к соседним сторонам \ (QP \) и \ (QR \), то более общие утверждения теоремы 1.13 о том, что формулы верны для \ (\ textit {any} \) смежные стороны не обосновываются. Если бы мы выбрали \ (PQ \) и \ (PR \) в качестве соседних сторон, мы получили бы другой \ (\ textit {formula} \) для области, но можно показать (см. Упражнение 26), что разные формулы дадут одно и то же значение, поэтому выбор соседних сторон действительно не имеет значения, и теорема 1.13 действительно.

    Теорема 1.13 упрощает вычисление площади треугольника в трехмерном пространстве, чем с использованием традиционных геометрических методов.

    Пример 1.9

    Вычислите площадь треугольника \ (\ треугольник PQR \), где \ (P = (2,4, -7) \), \ (Q = (3,7,18) \) и \ (R = (-5,12,8) \).

    Рисунок 1.4.4 Решение

    Пусть \ (\ textbf {v} = \ overrightarrow {PQ} \) и \ (\ textbf {w} = \ overrightarrow {PR} \), как на рисунке 1. 2} = \ frac {1} {2} \, \ sqrt {60966} \\ [4pt]
    A & \ приблизительно 123.{3} \) такая, что координата \ (z \) всегда равна \ (0 \). Таким образом, мы можем написать \ (\ textbf {v} = (-3, -1,0) \) и \ (\ textbf {w} = (1,2,0) \). Тогда площадь \ (A \) \ (PQRS \) равна

    \ begin {align *}
    A & = \ norm {\ textbf {v} \ times \ textbf {w}} = \ norm {(- 3, -1,0) \ times (1,2,0)} \\ [4pt]
    & = \ norm {((- 1) (0) — (0) (2), (0) (1) — (-3) (0), (-3) (2) — (-1) (1))} \\ [4pt]
    & = \ norm {(0,0, -5)} \\ [4pt]
    A & = 5
    \ end {align *}

    Следующая теорема суммирует основные свойства векторного произведения.{3} \) и скаляр \ (k \), имеем

    1. \ (\ textbf {v} \ times \ textbf {w} = — \ textbf {w} \ times \ textbf {v} \) Антикоммутативный закон
    2. \ (\ textbf {u} \ times (\ textbf {v} + \ textbf {w}) = \ textbf {u} \ times \ textbf {v} + \ textbf {u} \ times \ textbf {w} \ ) Закон о распределении
    3. \ ((\ textbf {u} + \ textbf {v}) \ times \ textbf {w} = \ textbf {u} \ times \ textbf {w} + \ textbf {v} \ times \ textbf {w} \ ) Распределительный закон
    4. \ ((k \ textbf {v}) \ times \ textbf {w} = \ textbf {v} \ times (k \ textbf {w}) = k (\ textbf {v} \ times \ textbf {w}) \) Ассоциативный закон
    5. \ (\ textbf {v} \ times \ textbf {0} = \ textbf {0} = \ textbf {0} \ times \ textbf {v} \)
    6. \ (\ textbf {v} \ times \ textbf {v} = \ textbf {0} \)
    7. \ (\ textbf {v} \ times \ textbf {w} = \ textbf {0} \) тогда и только тогда, когда \ (\ textbf {v} \ parallel \ textbf {w} \)

    Проба

    Доказательства свойств (b) — (f) несложны.Мы докажем части (a) и (g), а остальное предоставим читателю в качестве упражнений.

    Рисунок 1.4.6

    (a) По определению перекрестного произведения и скалярного умножения мы имеем:
    \ begin {align *}
    \ textbf {v} \ times \ textbf {w} & = (v_ {2 } w_ {3} — v_ {3} w_ {2},
    v_ {3} w_ {1} — v_ {1} w_ {3}, v_ {1} w_ {2} — v_ {2} w_ {1 }) \\ [4pt]
    & = — (v_ {3} w_ {2} — v_ {2} w_ {3},
    v_ {1} w_ {3} — v_ {3} w_ {1}, v_ {2} w_ {1} — v_ {1} w_ {2}) \\ [4pt]
    & = — (w_ {2} v_ {3} — w_ {3} v_ {2},
    w_ {3} v_ {1} — w_ {1} v_ {3}, w_ {1} v_ {2} — w_ {2} v_ {1}) \\ [4pt]
    & = — \ textbf {w} \ times \ textbf {v}
    \ end {align *}
    Обратите внимание, что здесь сказано, что \ (\ textbf {v} \ times \ textbf {w} \) и \ (\ textbf {w} \ times \ textbf {v} \) имеют той же величины, но в противоположном направлении (см. рисунок 1.4.6).

    (g) Если \ (\ textbf {v} \) или \ (\ textbf {w} \) равно \ (\ textbf {0} \), то \ (\ textbf {v} \ times \ textbf { w} = \ textbf {0} \) по части (e) и либо \ (\ textbf {v} = \ textbf {0} = 0 \ textbf {w} \), либо \ (\ textbf {w} = \ textbf {0} = 0 \ textbf {v} \), поэтому \ (\ textbf {v} \) и \ (\ textbf {w} \) являются скалярными кратными, т.е. они параллельны.
    Если и \ (\ textbf {v} \), и \ (\ textbf {w} \) отличны от нуля, а \ (\ theta \) — угол между ними, то по формуле (1.11) \ (\ textbf { v} \ times \ textbf {w} = \ textbf {0} \) тогда и только тогда, когда \ (\ norm {\ textbf {v}} \, \ norm {\ textbf {w}} \, \ sin \ theta = 0 \), что верно тогда и только тогда, когда \ (\ sin \ theta = 0 \) (поскольку \ (\ norm {\ textbf {v}}> 0 \) и \ (\ norm {\ textbf {w}} > 0) \).{\ circ} \) тогда и только тогда, когда \ (\ textbf {v} \ parallel \ textbf {w} \).

    \ (\ квадрат \)

    Пример 1.11

    Добавляем к примеру 1.7, у нас есть

    \ [\ nonumber \ textbf {i} \ times \ textbf {j} = \ textbf {k}, \ quad \ textbf {j} \ times \ textbf {k} = \ textbf {i}, \ quad \ textbf {k} \ times \ textbf {i} = \ textbf {j} \]
    \ [\ nonumber \ textbf {j} \ times \ textbf {i} = — \ textbf {k} , \ quad \ textbf {k} \ times \ textbf {j} = — \ textbf {i}, \ quad \ textbf {i} \ times \ textbf {k} = — \ textbf {j} \]
    \ [\ nonumber \ textbf {i} \ times \ textbf {i} = \ textbf {j} \ times \ textbf {j} = \ textbf {k} \ times \ textbf {k} = \ textbf {0} \]

    Вспомните из геометрии, что \ (\ textit {параллелепипед} \) — это трехмерное тело с 6 гранями, все из которых являются параллелограммами.{3} \) представляют собой смежные стороны параллелепипеда \ (P \), причем \ (\ textbf {u}, \ textbf {v}, \ textbf {w} \) образуют правостороннюю систему, как на рисунке 1.4. .7. Докажите, что объем \ (P \) — это \ (\ textit {тройное скалярное произведение} \) \ (\ textbf {u} \ cdot (\ textbf {v} \ times \ textbf {w}) \).

    Рисунок 1.4.7 Параллелепипед \ (P \)

    Решение

    Напомним, что объем \ (\ text {vol} (P) \) параллелепипеда \ (P \) равен площади \ (A \) базового параллелограмма, умноженной на высоту \ (h \). По теореме 1.13 (b), площадь \ (A \) базового параллелограмма равна \ (\ norm {\ textbf {v} \ times \ textbf {w}} \). И мы видим, что, поскольку \ (\ textbf {v} \ times \ textbf {w} \) перпендикулярен базовому параллелограмму, определяемому \ (\ textbf {v} \) и \ (\ textbf {w} \), тогда высота \ (h \) равна \ (\ norm {\ textbf {u}} \, \ cos \ theta \), где \ (\ theta \) — угол между \ (\ textbf {u} \) и \ (\ textbf {v} \ times \ textbf {w} \). По теореме 1.6 мы знаем, что

    \ [\ nonumber \ begin {align} \ cos \ theta & = \ dfrac {\ textbf {u} \ cdot (\ textbf {v} \ times \ textbf {w})} {\ norm {\ textbf {u} } \, \ norm {\ textbf {v} \ times \ textbf {w}}}.\ text {Следовательно,} \\ [4pt] \ nonumber \ text {vol} (P) & = A \, h \\ [4pt] \ nonumber & = \ norm {\ textbf {v} \ times \ textbf {w} } \, \ dfrac {\ norm {\ textbf {u}} \, \ textbf {u} \ cdot (\ textbf {v} \ times \ textbf {w})} {\ norm {\ textbf {u}} \ , \ norm {\ textbf {v} \ times \ textbf {w}}} \\ [4pt] \ nonumber & = \ textbf {u} \ cdot (\ textbf {v} \ times \ textbf {w}) \\ [ 4pt] \ end {align} \]

    В примере 1.12 высота \ (h \) параллелепипеда равна \ (\ norm {\ textbf {u}} \, \ cos \ theta \), а не \ (- \ norm {\ textbf {u}} \ , \ cos \ theta \), поскольку вектор \ (\ textbf {u} \) находится на той же стороне плоскости базового параллелограмма, что и вектор \ (\ textbf {v} \ times \ textbf {w} \) ( так что \ (\ cos \ theta> 0 \)).Поскольку объем одинаков, независимо от того, какое основание и высоту мы используем, повторение тех же шагов с использованием базы, определяемой \ (\ textbf {u} \) и \ (\ textbf {v} \) (поскольку \ (\ textbf {w} \) находится на той же стороне плоскости
    этой базы, что и \ (\ textbf {u} \ times \ textbf {v} \)), объем равен \ (\ textbf {w} \ cdot (\ textbf { u} \ times \ textbf {v}) \). {3} \),

    \ [\ textbf {u} \ cdot (\ textbf {v} \ times \ textbf {w}) = \ textbf {w} \ cdot (\ textbf {u} \ times \ textbf {v}) = \ textbf {v} \ cdot (\ textbf {w } \ times \ textbf {u}) \ label {Eq1.{3} \),

    \ [\ textbf {u} \ times (\ textbf {v} \ times \ textbf {w}) = (\ textbf {u} \ cdot \ textbf {w}) \ textbf {v } — (\ textbf {u} \ cdot \ textbf {v}) \ textbf {w} \ label {Eq1.13} \]

    Изучение формулы из теоремы 1.16 дает некоторое представление о геометрии тройного векторного произведения. В правой части уравнения \ ref {Eq1.13} мы видим, что \ (\ textbf {u} \ times (\ textbf {v} \ times \ textbf {w}) \) является скалярной комбинацией \ (\ textbf {v} \) и \ (\ textbf {w} \) и, следовательно, лежит в плоскости, содержащей \ (\ textbf {v} \) и \ (\ textbf {w} \) (i.е. \ (\ textbf {u} \ times (\ textbf {v} \ times \ textbf {w}) \), \ (\ textbf {v} \) и \ (\ textbf {w} \) являются \ (\ textbf {coplanar} \)). Это имеет смысл, поскольку по теореме 1.11 \ (\ textbf {u} \ times (\ textbf {v} \ times \ textbf {w}) \) перпендикулярно как \ (\ textbf {u} \), так и \ ( \ textbf {v} \ times \ textbf {w} \). В частности, перпендикулярность \ (\ textbf {v} \ times \ textbf {w} \) означает, что \ (\ textbf {u} \ times (\ textbf {v} \ times \ textbf {w}) \) лежит в плоскости, содержащей \ (\ textbf {v} \) и \ (\ textbf {w} \), поскольку эта плоскость перпендикулярна \ (\ textbf {v} \ times \ textbf {w} \).Но как тогда \ (\ textbf {u} \ times (\ textbf {v} \ times \ textbf {w}) \) также перпендикулярно \ (\ textbf {u} \), который может быть любым вектором? Следующий пример может помочь увидеть, как это работает.

    Пример 1.13

    Найдите \ (\ textbf {u} \ times (\ textbf {v} \ times \ textbf {w}) \) для \ (\ textbf {u} = (1, 2, 4) \), \ (\ textbf {v} = (2, 2, 0) \), \ (\ textbf {w} = (1, 3, 0) \).

    Решение

    Поскольку \ (\ textbf {u} \ cdot \ textbf {v} = 6 \) и \ (\ textbf {u} \ cdot \ textbf {w} = 7 \), то

    \ begin {align *}
    \ textbf {u} \ times (\ textbf {v} \ times \ textbf {w}) & = (\ textbf {u} \ cdot \ textbf {w}) \ textbf {v} — (\ textbf {u} \ cdot \ textbf {v}) \ textbf {w} \\ [4pt]
    & = 7 \, (2, 2, 0) — 6 \, (1, 3, 0) = (14, 14, 0) — (6, 18, 0) \\ [4pt]
    & = (8, -4, 0)
    \ end {align *}

    Обратите внимание, что \ (\ textbf {v} \) и \ (\ textbf {w} \) лежат в \ (xy \) — плоскости, и что \ (\ textbf {u} \ times (\ textbf {v} \ times \ textbf {w}) \) тоже лежит в этой плоскости.Кроме того, \ (\ textbf {u} \ times (\ textbf {v} \ times \ textbf {w}) \) перпендикулярно как \ (\ textbf {u} \), так и \ (\ textbf {v} \ times \ textbf {w} = (0, 0, 4) \) (см. рисунок 1.4.8).

    Рисунок 1.4.8

    Для векторов \ (\ textbf {v} = v_ {1} \ textbf {i} + v_ {2} \ textbf {j} + v_ {3} \ textbf {k} \) и \ (\ textbf {w} = w_ {1} \ textbf {i} + w_ {2} \ textbf {j} + w_ {3} \ textbf {k} \) в компонентной форме, перекрестное произведение записывается как \ (\ textbf {v} \ times \ textbf {w} = (v_ {2} w_ {3} — v_ {3} w_ {2}) \ textbf {i} + (v_ {3} w_ {1} — v_ {1} w_ {3 }) \ textbf {j} + (v_ {1} w_ {2} — v_ {2} w_ {1}) \ textbf {k} \).Часто проще использовать компонентную форму для перекрестного произведения, потому что она может быть представлена ​​как \ (\ textit {определитель} \). Мы не будем углубляться в теорию детерминант; мы просто рассмотрим то, что необходимо для наших целей.

    A 2 \ (\ times \) 2 матрица } — это массив из двух строк и двух столбцов скаляров, записанный как

    \ [\ nonumber \ begin {bmatrix} a & b \\ [4pt] c & d \ end {bmatrix} \ text {или} \ begin {pmatrix} a & b \\ [4pt] c & d \ end { pmatrix} \]

    где \ (a, b, c, d \) — скаляры.\ (\ Textbf {определитель} \) такой матрицы, записанный как

    \ [\ nonumber \ begin {vmatrix} a & b \\ [4pt] c & d \ end {vmatrix} \ text {или} \ det \ begin {bmatrix} a & b \\ [4pt] c & d \ конец {bmatrix}, \]

    — это скаляр, определяемый следующей формулой:

    \ [\ nonumber \ begin {vmatrix} a & b \\ [4pt] c & d \ end {vmatrix} = ad — bc \]

    Может быть полезно запомнить эту формулу как произведение скаляров на нисходящей диагонали минус произведение скаляров на восходящей диагонали.

    Пример 1.14

    \ [\ nonumber \ begin {vmatrix} 1 & 2 \\ [4pt] 3 & 4 \ end {vmatrix} = (1) (4) — (2) (3) = 4 — 6 = -2 \]

    A \ (3 \ times 3 \) матрица — это массив из трех строк и трех столбцов скаляров, записанный как

    \ [\ nonumber \ begin {bmatrix} a_ {1} & a_ {2} & a_ {3} \\ [4pt]
    b_ {1} & b_ {2} & b_ {3} \\ [4pt]
    c_ {1} & c_ {2} & c_ {3}
    \ end {bmatrix}
    \ text {или}
    \ begin {pmatrix} a_ {1} & a_ {2} & a_ {3} \\ [4pt ]
    b_ {1} & b_ {2} & b_ {3} \\ [4pt]
    c_ {1} & c_ {2} & c_ {3}
    \ end {pmatrix}, $$
    и его определитель задается формулой:
    \ [\ begin {vmatrix} a_ {1} & a_ {2} & a_ {3} \\ [4pt]
    b_ {1} & b_ {2} & b_ {3} \\ [ 4pt]
    c_ {1} & c_ {2} & c_ {3}
    \ end {vmatrix}
    = a_ {1} \ begin {vmatrix} b_ {2} & b_ {3} \\ [4pt] c_ { 2} & c_ {3} \ end {vmatrix} \; — \;
    a_ {2} \ begin {vmatrix} b_ {1} & b_ {3} \\ [4pt] c_ {1} & c_ {3} \ end {vmatrix} \; + \;
    a_ {3} \ begin {vmatrix} b_ {1} & b_ {2} \\ [4pt] c_ {1} & c_ {2} \ end {vmatrix} \]

    Один из способов запомнить приведенную выше формулу заключается в следующем: умножьте каждый скаляр в первой строке на определитель матрицы \ (2 \ times 2 \), которая остается после удаления строки и столбца, содержащих этот скаляр, затем суммируйте эти произведения вверх, чередуя знаки плюса и минуса перед каждым (начиная с плюса).

    Пример 1.15

    \ [\ nonumber \ left | \ begin {array} {rrr} 1 & 0 & 2 \\ [4pt] 4 & -1 & 3 \\ [4pt] 1 & 0 & 2 \ end {array} \ right |
    = 1 \ left | \ begin {array} {rr} -1 & 3 \\ [4pt] 0 & 2 \ end {array} \ right | \; — \;
    0 \ left | \ begin {array} {rr} 4 & 3 \\ [4pt] 1 & 2 \ end {array} \ right | \; + \;
    2 \ left | \ begin {array} {rr} 4 & -1 \\ [4pt] 1 & 0 \ end {array} \ right |
    = 1 (-2-0) — 0 (8-3) + 2 (0 + 1) = 0 \]

    Мы определили определитель как скаляр, полученный из алгебраических операций над скалярными элементами в матрице.Однако, если мы поместим три \ (\ textit {vectors} \) в первую строку матрицы \ (3 \ times 3 \), тогда определение все равно будет иметь смысл, поскольку мы будем выполнять скалярное умножение этих трех векторов ( они будут умножены на скалярные детерминанты \ (2 \ умножить на 2 \), как и раньше). Это дает нам определитель, который теперь является вектором, и позволяет нам записать перекрестное произведение \ (\ textbf {v} = v_ {1} \ textbf {i} + v_ {2} \ textbf {j} + v_ {3 } \ textbf {k} \) и \ (\ textbf {w} = w_ {1} \ textbf {i} + w_ {2} \ textbf {j} + w_ {3} \ textbf {k} \) в качестве определитель:

    \ begin {align *}
    \ nonumber \ textbf {v} \ times \ textbf {w} =
    \ begin {vmatrix} \ textbf {i} & \ textbf {j} & \ textbf {k} \\ [4pt ] v_ {1} & v_ {2} & v_ {3} \\ [4pt]
    w_ {1} & w_ {2} & w_ {3}
    \ end {vmatrix} & = \ begin {vmatrix} v_ { 2} & v_ {3} \\ [4pt] w_ {2} & w_ {3} \ end {vmatrix} \ textbf {i} \; — \;
    \ begin {vmatrix} v_ {1} & v_ {3} \\ [4pt] w_ {1} & w_ {3} \ end {vmatrix} \ textbf {j} \; + \;
    \ begin {vmatrix} v_ {1} & v_ {2} \\ [4pt] w_ {1} & w_ {2} \ end {vmatrix} \ textbf {k} \\ [4pt] \ nonumber
    & = ( v_ {2} w_ {3} — v_ {3} w_ {2}) \ textbf {i} + (v_ {3} w_ {1} — v_ {1} w_ {3}) \ textbf {j} +
    (v_ {1} w_ {2} — v_ {2} w_ {1}) \ textbf {k}
    \ end {align *}

    Пример 1.16

    Пусть \ (\ textbf {v} = 4 \, \ textbf {i} — \ textbf {j} + 3 \, \ textbf {k} \) и \ (\ textbf {w} = \ textbf {i} + 2 \, \ textbf {k} \). Тогда

    \ [\ nonumber \ textbf {v} \ times \ textbf {w} =
    \ left | \ begin {array} {rrr}
    \ textbf {i} & \ textbf {j} & \ textbf {k} \\ [4pt]
    4 & -1 & 3 \\ [4pt]
    1 & 0 & 2
    \ end {array} \ right |
    = \ left | \ begin {array} {rr} -1 & 3 \\ [4pt] 0 & 2 \ end {array} \ right | \ textbf {i} \; — \;
    \ left | \ begin {array} {rr} 4 & 3 \\ [4pt] 1 & 2 \ end {array} \ right | \ textbf {j} \; + \;
    \ left | \ begin {array} {rr} 4 & -1 \\ [4pt] 1 & 0 \ end {array} \ right | \ textbf {k}
    = -2 \, \ textbf {i} — 5 \, \ textbf {j} + \ textbf {k} \]

    Скалярное тройное произведение также можно записать как определитель.Фактически, в примере 1.12 следующая теорема дает альтернативное определение определителя матрицы \ (3 \ times 3 \) как объема параллелепипеда, смежные стороны которого являются строками матрицы и образуют правую систему (левосторонняя система даст отрицательную громкость). {3} \):
    \ [\ textbf {u} \ cdot (\ textbf {v} \ times \ textbf {w}) =
    \ begin {vmatrix} u_ {1} & u_ {2} & u_ {3} \\ [4pt]
    v_ {1} & v_ {2} & v_ {3} \\ [4pt]
    w_ {1} & w_ {2} & w_ {3}
    \ end {vmatrix} \]

    Пример 1.17

    Найдите объем параллелепипеда со смежными сторонами \ (\ textbf {u} = (2, 1, 3) \), \ (\ textbf {v} = (-1, 3, 2) \), \ (\ textbf {w} = (1, 1, -2) \) (см. рисунок 1.4.9).

    Рисунок 1.4.9 \ (P \)

    Решение

    По теореме 1.15 объем \ (\ text {vol} (P) \) параллелепипеда \ (P \) является абсолютной величиной скалярного тройного произведения трех смежных сторон (в любом порядке). По теореме 1.17,

    \ begin {align *}
    \ nonumber \ textbf {u} \ cdot (\ textbf {v} \ times \ textbf {w}) & =
    \ left | \ begin {array} {rrr}
    2 & 1 & 3 \\ [4pt]
    -1 & 3 & 2 \\ [4pt]
    1 & 1 & -2
    \ end {array} \ right | \\ [4pt] \ nonumber
    & = 2 \ left | \ begin {array} {rr} 3 & 2 \\ [4pt] 1 & -2 \ end {array} \ right | \; — \;
    1 \ left | \ begin {array} {rr} -1 & 2 \\ [4pt] 1 & -2 \ end {array} \ right | \; + \;
    3 \ left | \ begin {array} {rr} -1 & 3 \\ [4pt] 1 & 1 \ end {array} \ right | \\ [4pt] \ nonumber
    & = 2 (-8) — 1 (0) + 3 (-4) = -28 \ text {, поэтому} \\ [4pt] \ nonumber
    \ text {vol} (P) & = | -28 | = 28. {3} \ ).

    Решение

    Пусть \ (\ textbf {x} = \ textbf {u} \ times \ textbf {v} \). Тогда

    \ begin {align *}
    (\ textbf {u} \ times \ textbf {v}) \ cdot (\ textbf {w} \ times \ textbf {z}) & =
    \ textbf {x} \ cdot (\ textbf {w} \ times \ textbf {z}) \\ [4pt]
    & = \ textbf {w} \ cdot (\ textbf {z} \ times \ textbf {x}) \ text {(по формуле \ ref { Eq1.1.2})} \\ [4pt]
    & = \ textbf {w} \ cdot (\ textbf {z} \ times (\ textbf {u} \ times \ textbf {v})) \\ [4pt]
    & = \ textbf {w} \ cdot ((\ textbf {z} \ cdot \ textbf {v}) \ textbf {u} —
    (\ textbf {z} \ cdot \ textbf {u}) \ textbf {v} ) \ text {(по теореме 1.16)} \\ [4pt]
    & = (\ textbf {z} \ cdot \ textbf {v}) (\ textbf {w} \ cdot \ textbf {u}) —
    (\ textbf {z} \ cdot \ textbf {u}) (\ textbf {w} \ cdot \ textbf {v}) \\ [4pt]
    & = (\ textbf {u} \ cdot \ textbf {w}) (\ textbf {v} \ cdot \ textbf {z}) —
    (\ textbf {u} \ cdot \ textbf {z}) (\ textbf {v} \ cdot \ textbf {w}) \ text {(по коммутативности скалярного произведения).} \\ [4pt]
    & = \ begin {vmatrix}
    \ textbf {u} \ cdot \ textbf {w} & \ textbf {u} \ cdot \ textbf {z} \\ [4pt]
    \ textbf {v} \ cdot \ textbf {w} & \ textbf {v} \ cdot \ textbf {z} \ end {vmatrix}
    \ end {align *}

    Авторы и авторство

    Когда два вектора перпендикулярны, их класс взаимного произведения 12 математика CBSE

    Подсказка: Сначала мы определяем термины вектор, векторное произведение и перпендикуляр и решаем данную задачу дальше.0} \ Rightarrow \ left | A \ right | \ left | B \ right | $
    Следовательно, когда два заданных вектора перпендикулярны, их перекрестное произведение не равно нулю, а скалярное произведение равно нулю.

    Примечание: Коммутативность сохраняется в скаляре $ ab = ba $ и не выполняется в векторных произведениях, как $ a \ times b \ ne b \ times a $
    Параллельные прямые не пересекаются ни с одной из других линии, в отличие от перпендикулярных линий.
    Параллельные прямые — это, скорее, прямые в данной плоскости.0}}} \ Rightarrow \ dfrac {0} {1} = 0 $

    Что происходит, когда перекрестное произведение двух векторов равно 0? — Mvorganizing.org

    Что происходит, когда произведение двух векторов равно 0?

    Если два вектора имеют одинаковое направление или совершенно противоположные направления друг от друга (т. Е. Они не являются линейно независимыми), или если любой из них имеет нулевую длину, то их перекрестное произведение равно нулю.

    Когда два ненулевых вектора A и B перпендикулярны друг другу?

    Когда два ненулевых вектора и b перпендикулярны друг другу, Когда две силы величиной P и Q перпендикулярны друг другу, их равнодействующая имеет величину R.Когда они расположены под углом 180∘ друг к другу, их результирующая величина R√2.

    Что означает параллельность двух векторов?

    Два вектора u и v называются параллельными, если они имеют одинаковое или противоположное направление. Это означает, что каждый является скалярным, кратным другому: для некоторого ненулевого скаляра s v = su и поэтому u = v.

    Что произойдет, если два вектора перпендикулярны?

    Если два вектора перпендикулярны друг другу, то их скалярное произведение равно нулю.

    Как узнать, перпендикулярны ли два вектора?

    Если два вектора перпендикулярны, то их скалярное произведение равно нулю. Перекрестное произведение двух векторов определяется как A × B = (a2_b3 — a3_b2, a3_b1 — a1_b3, a1_b2 — a2 * b1). Перекрестное произведение двух непараллельных векторов — это вектор, перпендикулярный им обоим.

    Как узнать, что что-то параллельно или ортогонально?

    Два вектора не ортогональны; мы знаем это, потому что у ортогональных векторов есть скалярное произведение, равное нулю.Определите, параллельны ли два вектора, найдя угол между ними. Если бы они были параллельны, угол был бы 0 ° или 180 °, следовательно, два вектора не параллельны.

    Почему кросс-произведение дает перпендикулярный вектор?

    Представьте себе плоскость, содержащую два вектора a и b, а угол от a до b равен θ, а векторное произведение a и b равно || a || || b || грех (θ). Это потому, что когда вы переворачиваете плоскость, перекрестное произведение полностью переворачивается, что означает, что оно перпендикулярно плоскости.

    Перепендикулярно ли векторное произведение обоим векторам?

    Перекрестное произведение двух векторов всегда перпендикулярно плоскости, определяемой этими двумя векторами. Затем разделите произведение на его величину, чтобы получить единичный вектор.

    Является ли перекрестное произведение когда-либо коммутативным?

    Примечание. Перекрестные произведения не коммутативны. То есть u × v ≠ v × u. Векторы u × v и v × u имеют одинаковую величину, но направлены в противоположные стороны.

    Каково значение скалярного произведения двух перпендикулярных векторов? — Реабилитационная робототехника.нетто

    Каково значение скалярного произведения двух перпендикулярных векторов?

    Максимальное значение для скалярного произведения возникает, когда два вектора параллельны друг другу (вся «сила» от обоих векторов направлена ​​в одном направлении), но когда два вектора перпендикулярны друг другу, значение скалярного произведения равен 0 (один вектор имеет нулевую силу, направленную в направлении другого.

    Что такое скалярное произведение двух перпендикулярных векторов A и B?

    Ответ.следовательно, скалярное произведение двух взаимно перпендикулярных векторов будет НУЛЬ.

    Почему скалярное произведение двух перпендикулярных векторов равно нулю?

    Но почему мы так определили ортогональность? который умножает длину первого вектора на длину второго вектора на косинус угла между двумя векторами. А угол между двумя перпендикулярными векторами равен 90 °. Когда мы заменяем ø на 90 ° (cos 90 ° = 0), `a • b` становится равным нулю.

    Что делать, если скалярное произведение равно 0?

    Скалярное произведение двух векторов — это произведение их длин на косинус угла между ними.Если скалярное произведение равно 0, то либо длина одного, либо обоих равна 0, либо угол между ними равен 90 градусам.

    Два нулевых вектора перпендикулярны?

    Согласно этому определению перпендикулярности, линия, пересекающая нулевой вектор, отраженная относительно нулевого вектора, дает такую ​​же линию. Линия, отраженная, но не пересекающая нулевой вектор, дает параллельную линию. Следовательно, только векторы, которые пересекаются с данным нулевым вектором, перпендикулярны ему.

    Почему используется скалярное произведение?

    Скалярное произведение по существу говорит нам, какая часть вектора силы приложена в направлении вектора движения.Скалярное произведение также может помочь нам измерить угол, образованный парой векторов, и положение вектора относительно осей координат.

    Является ли скалярное произведение двух векторов скаляром?

    Скалярное произведение определяет компонент одного вектора в направлении другого, когда второй вектор нормализован. По сути, это скалярный множитель.

    Какое векторное произведение двух векторов приведем в двух примерах?

    Примером векторного произведения в физике является крутящий момент (момент силы — сила вращения).Сила, приложенная к рычагу, умноженная на его расстояние от точки опоры рычага O, представляет собой крутящий момент T, как показано на диаграмме.

    Какова формула произведения двух векторов?

    Если у вас есть два вектора a и b, то векторное произведение a и b равно c. Таким образом, это a × b на самом деле означает, что величина c = ab sinθ, где θ — угол между a и b, а направление c перпендикулярно скважине a, как b.

    Что представляет собой произведение двух векторов?

    Перекрестное произведение иногда называют векторным произведением двух векторов.Величина векторного произведения представляет собой площадь параллелограмма, стороны которого определяются двумя векторами, как показано на рисунке ниже.

    Почему кросс-произведение дает перпендикулярный вектор?

    Представьте себе плоскость, содержащую два вектора a и b, а угол от a до b равен θ, а векторное произведение a и b равно || a || || b || грех (θ). Это потому, что когда вы переворачиваете плоскость, перекрестное произведение полностью переворачивается, что означает, что оно перпендикулярно плоскости.

    Каким будет векторное тройное произведение трех векторов?

    Скалярное тройное произведение трех векторов a, b и c равно (a × b) ⋅c.Это скалярное произведение, потому что, как и скалярное произведение, оно дает одно число. (В этом смысле оно отличается от векторного произведения, которое является вектором.)

    Урок КАК доказать, что два вектора в координатной плоскости перпендикулярны

    КАК доказать, что два вектора в координатной плоскости перпендикулярны


    Предположим, что два вектора u и v заданы в координатной плоскости в компонентной форме u = (a, b) и v = (c, d).
    Как доказать, что эти векторы парпендикулярны?

    Два вектора u = (a, b) и v = (c, d) в координатной плоскости перпендикулярны тогда и только тогда, когда их скалярное произведение a * c + b * d равно нулю : А * с + Ь * г = 0.

    Пример 1
    Докажите, что векторы u = (,) и v = (,) перпендикулярны.

    Решение
    Скалярное произведение этих векторов равно

    ( u , v ) = * — * = 3 — 3 = 0.

    Поскольку скаляр векторов u и v равен нулю, эти векторы перпендикулярны.

    Пример 2
    Докажите, что векторы u = (,) и v = (,) перпендикулярны.

    Решение
    Скалярное произведение этих векторов равно

    ( u , v ) = + = — = — = 0.

    Поскольку скаляр векторов u и v равен нулю, эти векторы перпендикулярны.

    Для справки см. Урок «Перпендикулярные векторы в координатной плоскости» в теме Введение в векторы, сложение и масштабирование раздела Алгебра-II на этом сайте.

    Мои уроки по Dot-продукту на этом сайте
    — Введение в точечный продукт
    — Формула скалярного произведения векторов на плоскости через компоненты векторов
    — Точечное произведение векторов в координатной плоскости и угла между двумя векторами
    — Перпендикулярные векторы в координатной плоскости
    — Решенные задачи по скалярному произведению векторов и углу между двумя векторами.
    — Свойства скалярного произведения векторов в координатной плоскости
    — Формула угла между двумя векторами и формула косинусов разности двух углов

    Есть короткие уроки « КАК.. . Тип на точечном продукте:
    — КАК найти скалярное произведение двух векторов на плоскости
    — КАК найти скалярное произведение двух векторов в координатной плоскости
    — КАК найти угол между двумя векторами в координатной плоскости
    — КАК доказать, что два вектора в координатной плоскости перпендикулярны (этот урок)
    — КАК ДОКАЗАТЬ, что треугольник в координатной плоскости является прямоугольным треугольником
    — КАК проверить, является ли четырехугольник в координатной плоскости параллелограммом
    — КАК проверить, является ли четырехугольник в координатной плоскости прямоугольником
    — КАК проверить, является ли четырехугольник в координатной плоскости ромбом
    — КАК проверить, является ли четырехугольник в координатной плоскости квадратом

    Полный список моих уроков по точечному продукту с короткими аннотациями см. В файле ОБЗОР уроков по дот-продукту.

    Используйте этот файл / ссылку ALGEBRA-II — YOUR ONLINE TEXTBOOK для навигации по всем темам и урокам онлайн-учебника ALGEBRA-II.

    Ортогональный вектор — объяснение и примеры

    В области векторной геометрии мы рассмотрели почти все концепции векторов. Мы рассмотрели нормальные векторы, векторные уравнения, векторные скалярные произведения и многое другое. Но одна из самых важных концепций в этой области — понимание ортогонального вектора .

    Ортогональные векторы определяются как:

    «Два вектора называются ортогональными, если они перпендикулярны друг другу, и после выполнения анализа скалярного произведения результат, который они дают, равен нулю.”

    В этом разделе мы сосредоточимся на следующих областях:

    • Что такое ортогональный вектор?
    • Как найти ортогональный вектор?
    • Каковы свойства ортогонального вектора?
    • Примеры
    • Практические задачи


    Что такое ортогональный вектор?

    В математических терминах слово ортогональный означает направленный под углом 90 °. Два вектора u, v ортогональны, если они перпендикулярны, т.е.е., они образуют прямой угол, или если скалярный продукт, который они дают, равен нулю .

    Таким образом, мы можем сказать, что ,

    u⊥v или u · v = 0

    Следовательно, скалярное произведение используется для проверки того, направлены ли два вектора, которые наклонены рядом друг с другом, на угол 90 ° или нет.

    Если мы углубимся в свойства ортогонального вектора, мы узнаем, что нулевой вектор, который в основном является нулем, практически ортогонален каждому вектору.Мы можем проверить это, потому что u.0 = 0 для любого вектора и , нулевой вектор ортогонален каждому вектору. Это связано с тем, что нулевой вектор равен нулю и, очевидно, даст нулевой или нулевой результат при умножении на любое число или любой вектор.

    Два вектора, u и y, в пространстве внутреннего продукта, V, ортогональны, если их внутренний продукт равен нулю

    (u, y) = 0

    Теперь, когда мы знаем, что скалярное произведение — это главный ключ к выяснению, являются ли 2 вектора ортогональными или нет, давайте проведем несколько примеров для лучшего понимания.

    Пример 1

    Проверьте, ортогональны ли векторы a = i + 2j и b = 2i — j.

    Решение

    Для проверки того, являются ли 2 вектора ортогональными или нет, мы будем вычислять скалярное произведение этих векторов:

    ab = (1 · 2) + (2 · (-1))

    ab = 2 — 2

    ab = 0

    Следовательно, поскольку скалярное произведение равно 0, два вектора ортогональны.

    Пример 2

    Являются ли векторы a = (3, 2) и b = (7, -5} ортогональными?

    Решение

    Для проверки того, являются ли 2 вектора ортогональными или нет, мы будет вычислять скалярное произведение этих векторов:

    a.b = (3.7) + (7. (-5))

    a.b = 21 — 35

    a.b = -14

    Поскольку скалярное произведение этих двух векторов не равно нулю, эти векторы не ортогональны.

    Как найти ортогональный вектор?

    Мы уже обсуждали, что один из способов нахождения ортогональных векторов — это проверка их скалярного произведения. Если скалярное произведение дает нулевой ответ, очевидно, что перемножаемые векторы были на самом деле ортогональными или перпендикулярными.

    Общее, которое можно использовать в этом отношении, выглядит следующим образом:

    a.b = 0

    Эта концепция также может быть расширена в форме компонентов вектора.

    Общее уравнение в этом случае принимает следующий вид:

    ab = (ax.bx) + (ay.by)

    ab = 0

    Следовательно, основным требованием к ортогональности векторов является то, что они всегда должны предоставлять скалярный продукт, который дает нам нулевой результат.

    Но давайте рассмотрим и другие сценарии и методологии.

    Два перемножаемых вектора могут существовать в любой плоскости. Нет никаких ограничений на то, чтобы они были ограничены только двумерными плоскостями. Итак, давайте расширим наше исследование и на трехмерные плоскости.

    Ортогональный вектор в случае двумерной плоскости

    Большинство математических задач ограничиваются двумерными плоскостями. Такая плоскость имеет только 2 оси, а именно ось x и ось y.В разделе единичных векторов мы также обсудили, что эти оси также могут быть представлены в терминах единичных векторов; ось x в виде единичного вектора i и ось y в виде единичного вектора j.

    Теперь предположим, что есть 2 вектора с именами a и b , которые существуют в двухмерной плоскости. Мы должны засвидетельствовать, ортогональны ли эти два вектора друг другу или нет, то есть перпендикулярны друг другу.

    Мы пришли к выводу, что для проверки ортогональности мы оцениваем скалярное произведение векторов, существующих на плоскости. Итак, скалярное произведение векторов a и b будет примерно таким, как показано ниже:

    a.b = | a | x | b | x cosθ

    Если два вектора ортогональны или перпендикулярны, то угол θ между ними будет 90 °.

    Как мы знаем,

    cosθ = cos 90 °

    And,

    cos 90 ° = 0

    Итак, мы можем переписать уравнение скалярного произведения как:

    a.b = | a | x | b | x cos 90 °

    a.b = 0

    Мы также можем выразить это явление в терминах компонент вектора.

    a.b = ax.bx + ay.by

    И мы упоминали выше, что в терминах представления на основе единичных векторов; мы можем использовать символы i и j.

    Следовательно,

    a.b = ai.bi + aj.bj

    a.b = 0

    Следовательно, если скалярное произведение также дает ноль в случае умножения компонентов, то два вектора ортогональны.

    Пример 3

    Определите, ортогональны ли векторы a = (5, 4) и b = (8, -10) друг другу.

    Решение

    Для проверки того, являются ли 2 вектора ортогональными или нет, мы будем вычислять скалярное произведение этих векторов:

    ab = ai.bi + aj.bj

    ab = (5.8) + (4. — 10)

    ab = 40 — 40

    ab = 0

    Таким образом, доказано, что два вектора ортогональны по своей природе.

    Пример 4

    Определите, ортогональны ли векторы a = (2, 8) и b = (12, -3) друг другу.

    Решение:

    Для проверки того, являются ли 2 вектора ортогональными или нет, мы будем вычислять скалярное произведение этих векторов:

    ab = ai.bi + aj.bj

    ab = (2.12) + (8. -3)

    ab = 24 — 24

    ab = 0

    Таким образом, доказано, что два вектора ортогональны по своей природе.

    Ортогональный вектор в случае трехмерной плоскости

    Большинство реальных задач требует, чтобы векторы выходили в трехмерной плоскости. Когда мы говорим о трехмерных плоскостях, нас сопровождает другая ось, а именно ось z.

    В этом случае, с включением третьей оси, ось z будет состоять из 3 компонентов, каждая из которых направлена ​​вдоль своей соответствующей оси, если мы говорим, что любой вектор существует в трехмерной плоскости. В таком случае 3 компонента вектора в трехмерной плоскости будут x-составляющей, y-составляющей и z-составляющей.

    Если мы представим эти компоненты в терминах единичных векторов, то мы уже знаем, что для осей x и y мы используем символы i и j для представления их компонентов. Но теперь, когда у нас есть третья ось и одновременно третий компонент, нам нужно дополнительное третье представление.

    Итак, для этой третьей оси мы используем символ k для представления единичного вектора вдоль оси z.

    Теперь представьте, что 2 вектора существуют в трехмерной плоскости.Эти векторы, очевидно, будут иметь 3 компонента, и скалярное произведение таких векторов можно найти ниже:

    ab = ax.bx + ay.by + az.bz

    Или, в терминах единичных векторов i, j, и k :

    ab = ai.bi + aj.bj + ak.bk

    ab = 0

    Следовательно, если этот результат дает скалярное произведение 0, мы сможем заключить, что 2 вектора в трехмерной плоскости перпендикулярны или ортогональны по своей природе.

    Пример 5

    Проверьте, ортогональны ли векторы a = (2, 3, 1) и b = (3, 1, -9).

    Решение

    Чтобы проверить, являются ли эти два вектора ортогональными или нет, мы будем вычислять их скалярное произведение. Поскольку эти 2 вектора имеют 3 компонента, следовательно, они существуют в трехмерной плоскости.

    Итак, мы можем написать:
    ab = ai.bi + aj.bj + ak.bk

    Теперь, подставив значения в формулу:

    ab = (2.3) + (3.1) + (1. -9)

    ab = 6 + 3 -9

    ab = 0

    Поскольку скалярное произведение равно нулю, эти два вектора в трехмерной плоскости ортогональны по своей природе.

    Пример 6

    Определите, являются ли 2 вектора a = i + 2j и b = 2i -j + 10k ортогональными или нет.

    Решение

    Чтобы проверить, являются ли эти два вектора ортогональными или нет, мы будем вычислять их скалярное произведение. Поскольку эти 2 вектора имеют 3 компонента, следовательно, они существуют в трехмерной плоскости.

    Итак, мы можем написать:
    a.b = ai.bi + aj.bj + ak.bk

    Теперь, подставив значения в формулу:

    a.b = (1.2) + (2. -1) + (0.10)

    ab = 2-2 + 0

    ab = 0

    Поскольку скалярное произведение равно нулю, следовательно, эти 2 вектора в трехмерной плоскости равны ортогональные по своей природе.

    Пример 7

    Проверьте, ортогональны ли 2 вектора a = (2, 4, 1) и b = (2, 1, -8).

    Решение

    Чтобы проверить, являются ли эти два вектора ортогональными или нет, мы будем вычислять их скалярное произведение. Поскольку эти 2 вектора имеют 3 компонента, следовательно, они существуют в трехмерной плоскости.

    Итак, мы можем написать:

    ab = ai.bi + aj.bj + ak.bk

    Теперь, подставив значения в формулу:

    ab = (2.2) + (4.1) + (1 . -8)

    ab = 4 + 4 — 8

    ab = 0

    Поскольку скалярное произведение равно нулю, эти два вектора в трехмерной плоскости ортогональны по своей природе.

    Свойства ортогональных векторов

    Теперь, когда мы просмотрели всю необходимую информацию об ортогональных векторах и получили четкое представление о том, как проверить, являются ли векторы ортогональными или нет, давайте проанализируем некоторые свойства ортогональные векторы.

    Перпендикулярно по природе

    Векторы, которые считаются ортогональными, всегда будут перпендикулярными по своей природе и всегда будут давать скалярный продукт равным 0, поскольку перпендикулярность означает, что между ними будет угол 90 °.

    Нулевой вектор ортогонален

    Нулевой вектор всегда будет ортогонален каждому вектору, с которым существует нулевой вектор. Это связано с тем, что любой вектор при умножении на нулевой вектор всегда давал бы точечный продукт равным нулю.

    Перекрестное произведение ортогональных векторов

    Перекрестное произведение 2 ортогональных векторов никогда не может равняться нулю. Это связано с тем, что формула векторного произведения включает тригонометрическую функцию sin, а грех 90 ° всегда равен 1. Следовательно, векторное произведение ортогональных векторов никогда не будет равно 0.

    Практические задачи:
    1. Определите, ортогональны ли векторы (1, 2) и (2, -1).
    2. Определите, ортогональны ли векторы (1, 0, 3) и (4, 7, 4).
    3. Докажите, что произведение ортогональных векторов не равно нулю.

    Ответы
    1. Да
    2. Нет
    3. Докажите с помощью формулы взаимного произведения

    Все диаграммы построены с использованием GeoGebra.

    alexxlab

    Добавить комментарий

    Ваш адрес email не будет опубликован. Обязательные поля помечены *